--> -->
ADD TO FAVORITES!!!

SHARE YOUR FEEDBACK

About 36garhonline

36garhonline.blogspot.com is the complete EXAM Online information Portal of state Chhattisgarh and all over India,where one can easily search or can check information about all exams in Chhattisgarh.

Check EXAM PATTERN, Study material, previous year papers, Free Download books for technical Exam In RESOURCE Section.

Check Monthly Basis Current Affair News and preparation notes on Notes For GK section.

Chattisgarh State -India

Chhattisgarh is India's 26th state, covers an area of 135,133 sq km. of the total geographical area of nation India and located in Center of India.

Chhattisgarh state of India came into existence on November 1, 2000. Mother tongue of Chhattisgarh is Chattisgarhi.There are 18 districts (initially there was 15 districts) in the state.

6 States of India which are bordered with Chhattisgarh state -
# Bihar, Jharkhand and Uttar Pradesh to the north
# Andhra Pradesh to the south.
# Orissa State to the east.
And by the Madhya Pradesh to the west.

About CGPSC, Colleges, University, Admission in Chattisgarh

Chhattisgarh Public Service Commission, i.e. CGPSC examination held by Chhattisgarh State for graduate students and gazette posts, rajya seva posts. To say CGPSC is a Constitutional Body established to conduct examinations and recruit qualified candidates for appointment to the service of Government of Chhattisgarh.

Prospective candidates of CGPSC 2011 have to be cautious to enter accurate personal details for registration. Regular updates regarding advertisements and notifications of CGPSC 2011 can be accessed by creating a registered profile on the official website. To apply for an examination a candidate must go through the registration process. The CGPSC main exam will be held after preliminary exam for CGPSC 2011.

To know more about Chhattisgarh Lokseva Ayog, CGPSC Exam Result 2010 and Interview Schedule, Subjects of CGPSC Examinations, CGPSC Advertisement and CGPSC Syllabus visit our blog.

Solved Papers-GIC(General Insurance Corporation Of India)-Assistant Manager (Scale-I)

Saturday 29 October, 2011
Published by Roma Rahul Gupta for Chhattisgarh Online
Below are some sample paper for the said exam. Though the level of difficulty may vary in the exam. However, the basis and the pattern will remain the same. As not many exam of  Insurance have taken in the past so there no sure pattern, but believe me this question paper will have more or less same set of questions as below. We are also trying to chalk out more sample paper for other different section. They would be uploaded as soon as they are available. All the best and keep practicing. Hard and fast. And o remember that such exams are all about speed but speed should not come at the cost of accuracy.


General and Financial Awareness
(Held on 14-11-2010)


1. The present Cash Reserve Ratio is—

(A) 5%

(B) 5.5%


(C) 6%

(D) 6.5%

(E) None of these

Ans : (C)



2. One of the objectives of KYC (Know Your Customer) norms is—

(A) to give boost to bank deposits

(B) to safeguard banks advances

(C) to monitor transactions of suspicious nature


(D) to help income tax authorities to collect income tax

(E) None of these

Ans : (C)



3. Contribution to Prime Minister's relief fund enjoys Income Tax benefit up to—

(A) 50% under section 80G

(B) 75% under section 80G

(C) 100% under section 80 G

(D) 100% under section 88


(E) No exemption is available

Ans : (C)



4. Which of the following activities are expected to be performed by the Business correspondents ?

(A) Disbursal of small value credit

(B) Collection of small value deposits

(C) Sale of micro insurance /mutual fund products

(D) All the three above

(E) Only (B) and (C) above


Ans : (E)



5. Tax at source by banks is deducted on interest paid on term deposits in the interest amount in a financial year exceeds—

(A) Rs. 3,000

(B) Rs. 5,000

(C) Rs. 10,000

(D) Rs. 15,000

(E) There is no such provision

Ans : (B)






6. Securitisation and Reconstruction of Financial Assets and Enforcement of security Interest Act relates to—

(A) Sanction of loans

(B) Enhancement of loan limits

(C) Recovery of loans

(D) All the above

(E) None of above

Ans : (C)




7. MSMED Act is applicable to—

(A) Smalls enterprises only

(B) Medium enterprises only

(C) Micro enterprises only

(D) Micro, Small and Medium enterprises

(E) All enterprises irrespective of their size engaged in manufacturing activity

Ans : (D)



8. Money Laundering refers to—


(A) Conversion of assets into cash

(B) Conversion of Money which is illegally obtained

(C) Conversion of cash into gold

(D) Conversion of gold into cash

(E) None of the above

Ans : (B)



9. The Monetary and Credit Policy is reviewed by the RBI after a gap of—

(A) one year


(B) one month

(C) two years

(D) five years

(E) None of these

Ans : (E)



10. PPF account is opened for a period of—

(A) 5 years

(B) 10 years


(C) 15 years

(D) 20 years

(E) None of these

Ans : (C)



11. Yuan is the currency of—

(A) Japan

(B) China

(C) Indonesia


(D) Myanmar

(E) None of these

Ans : (B)



12. National savings certificate matures at the end of—

(A) Six years

(B) Three years

(C) Six and half years

(D) Five years


(E) Five and half years

Ans : (A)



13. Normally Bank accept Fixed Deposits for a maximum period of—

(A) 5 years

(B) 3 years

(C) 10 years

(D) 20 years

(E) Any number of years


Ans : (E)



14. Code of banks commitment to Micro and Small enterprises is prepared by—

(A) RBI

(B) SEBI

(C) FEDAI

(D) BCSBI

(E) Ministry of Small and Medium Enterprises

Ans : (E)




15. Which one of the following is not a salient feature of debit card ?

(A) No bad debts to banks and no suits for recovery

(B) No interest earning for banks

(C) Works like a normal withdrawal

(D) All the above

(E) 45 dayes credit is given to the card holder

Ans : (E)




16. IFRS stands for—

(A) International Financial Reporting standards

(B) Indian Financial Rating Standards

(C) International Financial Rating Standards

(D) All the three above

(E) None of the above

Ans : (A)



17. What is the present Repo Rate ?


(A) 5%

(B) 5•5%

(C) 6%

(D) 6•5%

(E) None of these

Ans : (E)



18. There are certain financial instruments whose prices are derived from the price of the underlying currency of interest rate or stocks etc. These are known as—


(A) Derivatives

(B) Securitisation

(C) Leasing

(D) Factoring

(E) Venture Capital Funding

Ans : (A)



19. What is the full form of ASBA ?

(A) Allotment supported by Blocked Amount


(B) Application supported by Blocked Amount

(C) Application supported by Bank Amount

(D) Allotment supported by Bank Account

(E) None of the above

Ans : (B)



20. Reverse Repo is used by RBI to—

(A) Inject liquidity

(B) Absorb liquidity


(C) Increase the liquidity with banking system

(D) Keep the liquidity at one level

(E) None of the above

Ans : (D)



21. Which of the following is not considered as lending under infrastructure sector ?

(A) A Highway project

(B) Construction of Educational Institution


(C) Construction of Hospital

(D) Laying down of petroleum pipelines

(E) None of the above

Ans : (E)



22. KYC guidelines have been framed on the recommendations/as per guidelines of—

(A) Reserve Bank of India

(B) Ministry of Finance

(C) Indian Banks Association


(D) Financial Action Task Force

(E) Ministry of Home affairs

Ans : (A)



23. The term ‘Power of Attorney’ refers to—

(A) Power of a person

(B) An authority to operate a Bank account

(C) An instrument by which a person is empowered to act for another person


(D) All of the above

(E) None of the above

Ans : (C)



24. What is the amount of compensation to be paid per day, as per RBI directives in case of failed ATM transactions ?

(A) Rs. 50

(B) Rs. 100

(C) Rs. 200

(D) Rs. 500


(E) It is at the discretion of each Bank

Ans : (B)



25. Financial Action Task Force has an office in India at which place ?

(A) Mumbai

(B) Chennai

(C) Kolkata

(D) New Delhi

(E) All the above places


Ans : (D)



26. Bridge loans refer to—

(A) Loans granted to contruction companies for construction of bridges

(B) Loan granted to PWD for construction of bridges over Rivers

(C) Interim finance allowed by banks to their customers pending disbursement of term loans by financial institutions

(D) All of the above

(E) None of the above

Ans : (C)




27. Payment of Demand Draft can be stopped by—

(A) Payee

(B) Holder

(C) Purchaser

(D) All of these

(E) None of these

Ans : (A)



28. Can any one file an appeal against the order passed by the Banking Ombudsman ? If so who is the Appellate Authority ?


(A) The Chairman of concerned Bank

(B) The Deputy Governor RBI

(C) Governor of RBI

(D) Finance Minister

(E) None of the above

Ans : (B)



29. For which one of the following reasons, the Government has approved a plan to infuse Rs. 15,000 crore capital into PSBs ?

(A) To boost their lending capacity


(B) To maintain minimum CRAR as per Basel II norms

(C) To maintain NPA provision coverage ratio

(D) To strengthen the Balance Sheet of banks

(E) None of these

Ans : (A)



30. What is monetary policy transmission ?

(A) It refers to monetary policy of Central Bank

(B) It refers to fiscal policy of Government


(C) It refers to various channels through which the monetary policy of a Central Bank alters prices or output in the real economy

(D) It refers to various channels through which the fiscal policy of the Government alters prices or output in the real economy

(E) None of these

Ans : (C)



31. On which one of the following issue IMF has supported monetary policy of India ?

(A) Tightening of monetary policy

(B) Stimulus for agriculture sector

(C) Concessions for foreign investment


(D) Introduction of GST

(E) None of these

Ans : (A)



32. The advantage of convenience in credit card operations is for—

(A) Customer

(B) Members Establishments

(C) Banks

(D) All the above


(E) None of the above

Ans : (D)



33. Under provisions of which one of the following Acts, CAs/CS have been told to report all suspicious fund trasnfers ?

(A) RBI Act

(B) Banking Regulation Act

(C) Indian Companies Act

(D) Unlawful Activities (Prevention) Act

(E) None of these


Ans : (D)



34. Bancassurance is—

(A) an insurance scheme to insure bank deposits

(B) an insurance scheme exclusively for the employees of banks

(C) a composite financial service offering both bank and insurance product

(D) a bank deposits scheme exclusively for employees of insurance companies

(E) None of the above

Ans : (C)




35. In the term STRIPS, the first letter ‘S’ denotes—

(A) Separate

(B) Small

(C) Special

(D) Savings

(E) None of these

Ans : (A)




36. Which of the following organizations, provide credit history of the borrowers ?

(A) CIBIL

(B) SEBI

(C) RBI

(D) CRISIL

(E) IBA

Ans : (A)



37. Loans/advances to farmers is treated as—


(A) Personal Loans

(B) Priority Sector Loan

(C) Business Loan

(D) Corporate Loan

(E) None of these

Ans : (B)



38. Which one of the following Organisations maintains CRR ?

(A) RBI


(B) SEBI

(C) NABARD

(D) IBA

(E) None of these

Ans : (A)



39. When the loan is granted for purchase of white goods it is called—

(A) Consumption loan

(B) White goods loan


(C) Consumer durable loan

(D) All the above

(E) None of the above

Ans : (C)



40. Which one of the following is the objective of Mahatma Gandhi National Rural Employment Guarantee Act ?

(A) To provide 100 days employment to people in rural areas

(B) To provide employment to educated youth

(C) To provide employment under KVIC schemes


(D) To create more valuable rural assets

(E) None of these

Ans : (A)



41. Structure of Basel II is based on how many pillars ?

(A) Two

(B) Three

(C) Four

(D) Eight


(E) Six

Ans : (B)



42. With which one of the following ‘Channel Financing’ is associated ?

(A) Retail Lending

(B) Corporate Lending

(C) SME Lending

(D) Supply Chain Finance


(E) None of these

Ans : (D)



43. Expand the term FRBM—

(A) Financial Responsibility and Business Management

(B) Fiscal Responsibility and Business Management

(C) Financial Responsibility and Budget Management

(D) Fiscal Responsibility and Budget Management

(E) None of these


Ans : (D)



44. A customer can approach Banking ombudsman if he does not get satisfactory response to his grievance from the bank within how many days ?

(A) 10 days

(B) 20 days

(C) 8 days

(D) 30 days

(E) 60 days

Ans : (D)




45. Which one of the following country is in talks with EU and International Monetary Fund, to exit from debt crisis ?

(A) Spain

(B) Turkey

(C) Portugal

(D) Greece

(E) Finland

Ans : (D)



46. For achieving 8•5 percent GDP growth in fiscal 2010-11, which one of the following should be percent growth in farm sector ?


(A) 2•0

(B) 2•5

(C) 3•0

(D) 4•0

(E) None of these

Ans : (C)




47. According to a report submitted by IMF in its World Economic Outlook, which one of the following countries will have highest percent GDP growth rate in 2011 ?

(A) China

(B) India

(C) Brazil

(D) Russia

(E) None of these

Ans : (A)



48. Which one of the following has given ‘Aadhaar’ as its new brand name ?


(A) UIDAI

(B) Sports Ministry, GOI

(C) Ministry of Tourism, GOI

(D) NHAI

(E) None of these

Ans : (A)



49. Which of the following will help poor to come out of their poverty ?

1. Good Health Service


2. Freedom from illiteracy

3. Optimum Sex Ratio

(A) Only 1

(B) Only 2

(C) Only 1 and 2

(D) Only 3

(E) All 1, 2 and 3

Ans : (E)




50. Which one of the following is per cent Bank Rate ?

(A) 4.0

(B) 4.5

(C) 5

(D) 3.33

(E) None of these

Ans : (E)


*************************************************
Reasoning Ability 
(Held on 9-5-2010) 


1. If the following numbers are rewritten by interchanging the digits in ten’s place and hundred’s place and then arranging them in the descending order. What will be the second digit of the newly formed fifth number from your right ?


479, 736, 895, 978, 389, 675

(A) 3

(B) 4

(C) 5

(D) 6

Ans : (C)



2. P is 60 m South-East of Q. R is 60 m North-East of Q. Then R is in which direction of P ?

(A) North


(B) North-East

(C) South

(D) South-East

Ans : (A)



Directions—(Q. 3–5) Read the following information for answering the questions that follow—

On a playing ground A, B, C, D and E are standing as described below facing the North.

(i) B is 50 metres to the right of D.


(ii) A is 60 metres to the South of B

(iii) C is 40 metres to the West of D.

(iv) E is 80 metres to the North of A.



3. If a boy walks from C, meets D followed by B, A and then E, how many metres has he walked if he has travelled the straight distance all through ?

(A) 120

(B) 150

(C) 170

(D) 230


Ans : (D)



4. What is the minimum distance (in metre approximately) between C and E ?

(A) 53

(B) 78

(C) 92

(D) 120

Ans : (C)



5. Who is to the South-East of the person who is to the left of D ?


(A) A

(B) B

(C) C

(D) E

Ans : (A)





6. A man was walking in the evening just before the sun set. His wife said that, his shadow fell on his right. If the wife was walking in the opposite direction of the man, then which direction the wife was facing ?

(A) North

(B) West


(C) South

(D) East

Ans : (C)



Directions—(Q. 7–11) In each of the following questions choose the set of numbers from the four alternative sets that is similar to the given set.



7. Given set : (4, 9, 18)

(A) (8, 14, 22)

(B) (10, 15, 25)


(C) (6, 12, 23)

(D) (12, 17, 26)

Ans : (D)



8. Given set : (10, 14, 17)

(A) (4, 11, 14)

(B) (9, 12, 15)

(C) (8, 13, 18)

(D) (6, 9, 12)


Ans : (A)



9. Given set : (7, 27, 55)

(A) (21, 35, 52)

(B) (18, 42, 65)

(C) (16, 40, 72)

(D) (13, 30, 58)

Ans : (C)



10. Given set : (39, 28, 19)


(A) (84, 67, 52)

(B) (52, 25, 17)

(C) (70, 49, 36)

(D) (65, 45, 21)

Ans : (A)



11. Given set : (246, 257, 358)

(A) (233, 343, 345)

(B) (273, 365, 367)


(C) (143, 226, 237)

(D) (145, 235, 325)

Ans : (A)



Directions—(Q. 12–16) Each question contains six or seven statements followed by four sets of combinations of three. Choose the set in which the statements are logically related.



12. (1) All books are having pages.

(2) All kings are having pages.

(3) All kings are books.


(4) Some heavy things are having pages.

(5) Some heavy things are books.

(6) Some books are heavy.

(7) Some heavy things are having pages.

(A) 1, 2, 3

(B) 6, 1, 4

(C) 4, 6, 1

(D) 1, 5, 7

Ans : (D)




13. (1) No athletes are vegetarians.

(2) All cricket players are athletes.

(3) Some athletes play cricket.

(4) Some cricket players are vegetarians.

(5) No cricket player is a vegetarian.

(6) All athletes are vegetarians.

(A) 1, 2, 5

(B) 3, 4, 1


(C) 1, 5, 2

(D) 2, 5, 6

Ans : (A)



14. (1) All grandmothers cook well.

(2) No man is a grandmother.

(3) Some men do not cook well.

(4) All those who cook well are men.

(5) No one who cooks well is a man.


(6) All those who cook well are grandmothers.

(7) Some men are not grandmothers.

(A) 2, 6, 5

(B) 2, 5, 6

(C) 1, 4, 2

(D) 6, 4, 7

Ans : (B)



15. (1) Looting is a crime.


(2) Some crooked people are criminals.

(3) All those involved in looting are criminals.

(4) Some crooked people are involved in looting.

(5) All criminals are looked down in society.

(6) Some crooked people are not criminals.

(A) 1, 4, 6

(B) 3, 6, 2

(C) 1, 2, 6

(D) 3, 4, 2


Ans : (D)



16. (1) Some women are those who are successful in life.

(2) Some men are those who have patience.

(3) No man is a woman.

(4) All those who have patience are successful in life.

(5) Some who are successful in life are men.

(6) Some men are not those are successful in life.

(A) 1, 3, 6


(B) 4, 2, 6

(C) 1, 5, 3

(D) 2, 4, 5

Ans : (B)



Directions—(Q. 17–21) Each of the questions below consists of a question and two statements numbered (I) and (II). You have to decide whether the data provided in the statements are sufficient to answer the question. Give answers—

(A) If the data in statement (I) alone are sufficient to answer the question, while the data in statement (II) alone are not sufficient to answer the question;

(B) If the data in statement (II) alone are sufficient to answer the question, while the data in statement (I) alone are not sufficient to answer the questions;


(C) If the data even in both statements (I) and (II) together are not sufficient to answer the question;

(D) If the data in both statement (I) and (II) together are necessary to answer the question.



17. In which direction is Mahatmaji’s statue facing ?

I. The statue is towards the northern end of the city.

II. The statue’s shadow falls towards East at 5 O’clock in the evening.

Ans : (C)




18. What is the total number of pupils in the final year class ?

I. The number of boys in the final year class is twice as much as the number of girls in that class.

II. The sum of the ages of all the pupils in the class is 399 years and their average age is 19 years.

Ans : (B)



19. Who is the tallest among A, B, C and D ?

I. A is taller than C.

II. B is taller than C and D.

Ans : (C)




20. How many Sundays are there in a particular month of a particular year ?

I. The month begins on Monday.

II. The month ends on Wednesday.

Ans : (D)



21. What is the total number of pages in this book ?

I. I counted 132 pages from the beginning of this book.

II. My wife counted 138 pages starting from the end of the same book.

Ans : (C)




Directions—(Q. 22–26) In each of the questions given below, there is a statement followed by three assumptions numbered I, II and III. An assumption is something supposed or taken for granted. You have to consider the statement and assumptions and then decide, which of the assumption(s) is/are implicit in the statement.



22. Statement : During pre-harvest kharif seasons, the government has decided to release vast quantity of foodgrains from FCI.

Assumptions :I. There may be a shortage of foodgrains in the market during this season.

II. The kharif crop may be able to replenish the stock of FCI.

III. There may be a demand from the farmers to procure kharif crop immediately after harvest.


(A) None is implicit

(B) Only I and II are implicit

(C) Only II and III are implicit

(D) All are implicit

Ans : (D)



23. Statement : To improve the employment situation in India, there is a need to recast the present educational system towards implementation of scientific discoveries in daily life.

Assumptions :I. The students after completing such education may be able to earn their livelihood.


II. This may bring meaning of education in the minds of the youth.

III. The state may earn more revenue as more and more people will engage themselves in self employment.

(A) Only I and II are implicit

(B) Only III is implicit

(C) Only I and III are implicit

(D) None is implicit

Ans : (A)



24. Statement : To increase profit, the oil exporting countries decided to reduce the production of crude by 5 million barrels per day.


Assumptions :I. The price of crude may increase due to less production.

II. The demand of crude may remain same in future.

III. Other countries may continue buying crude from these countries.

(A) All are implicit

(B) Only II and III are implicit

(C) Only I and II are implicit

(D) None is implicit

Ans : (C)




25. Statement : “We do not want you to see our product on newspaper, visit our shop to get a full view.” – an advertisement.

Assumptions :I. People generally decide to purchase any product after seeing the name in the advertisement.

II. Uncommon appeal may attract the customers.

III. People may come to see the product.


(A) All are implicit

(B) None is implicit

(C) Only II and III are implicit

(D) Only I and II are implicit

Ans : (A)



26. Statement : The Reserve Bank of India has directed the banks to refuse fresh loans to major defaulters.

Assumptions :I. The banks may still give loans to the defaulters.


II. The defaulters may repay the earlier loan to get fresh loan.

III. The banks may recover the bad loans through such harsh measures.

(A) All are implicit

(B) None is implicit

(C) Both II and III are implicit

(D) Both I and II are implicit

Ans : (C)



Directions—(Q. 27–31) In questions given below, statements 1 and 2 are followed by conclusions I and II. Taking the statements to be right although they may seem at variance with commonly accepted facts, mark your answers as under—


(A) If only conclusion I follows.

(B) If only conclusion II follows.

(C) If both I and II follows.

(D) Neither I nor II follows.



27. Statements :1. All hands are machines.

2. All machines are wheels.

Conclusions :I. All wheels are hands.


II. All hands are wheels.

Ans : (B)



28. Statements :1. Some buds are leaves.

2. Some leaves are red.

Conclusions :

I. Some buds are red.

II. Some leaves are not buds.


Ans : (B)



29. Statements :

1. Some stones are shells.

2. All shells are pearls.

Conclusions :I. Some stones are pearls.

II. All pearls are shells.

Ans : (A)




30. Statements :1. Brown is red and blue is green.

2. Green is pink and yellow is red.

Conclusions :

I. Yellow is brown.

II. Pink is blue.

Ans : (C)




31. Statements :

1. Merchants who do not own cars do not have bicycles either.

2. Those who do not have bicycles have tricycles.

Conclusions :I. Some merchants have only tricycles.

II. No one has both, the car and the tricycles.

Ans : (D)



Directions—(Q. 32–36) A number arrangement machine, when given a particular input, rearranges it following a particular rule. The following is the illustration of the input and the stages of arrangement.


Input : 245, 316, 436, 519, 868, 710, 689

Step I : 710, 316, 436, 519, 868, 245, 689

Step II : 710, 316, 245, 519, 868, 436, 689

Step III : 710, 316, 245, 436, 868, 519, 689

Step IV : 710, 316, 245, 436, 519, 868, 689


Step IV is the last step of input.



32. If 655, 436, 764, 799, 977, 572, 333 is the input which of the following steps will be ‘333, 436, 572, 655, 977, 764, 799’ ?

(A) II

(B) III

(C) IV

(D) I


Ans : (B)



33. How many steps will be required to get the final output from the following input ?

Input : 544, 653, 325, 688, 461, 231, 857

(A) 5

(B) 4

(C) 3

(D) 6


Ans : (A)



34. For the given input, which of the following will be third step ?

Input : 236, 522, 824, 765, 622, 463, 358

(A) 522, 236, 765, 824, 622, 463, 358

(B) 522, 622, 236, 824, 765, 463, 358

(C) 522, 622, 236, 765, 824, 463, 358

(D) 522, 622, 236, 463, 824, 765, 358


Ans : (C)



35. If following is the second step for an input, what will be the fourth step ?

Step II : 620, 415, 344, 537, 787, 634, 977

(A) 620, 415, 344, 537, 634, 787, 977

(B) 620, 415, 344, 634, 537, 787, 977

(C) 620, 415, 344, 634, 787, 537, 977

(D) Can’t be determined


Ans : (B)



36. Which of the following is the last step for the following input ?

Input : 473, 442, 735, 542, 367, 234, 549

(A) 234, 442, 542, 473, 735, 367, 549

(B) 234, 442, 542, 735, 473, 367, 549

(C) 234, 442, 542, 473, 367, 735, 549

(D) 234, 442, 542, 735, 367, 473, 549


Ans : (A)



Directions—(Q. 37–41) Read the following information carefully and answer the questions given below it—

(1) There is a group of six persons A, B, C, D, E and F in a family. They are Psychologist, Manager, Lawyer, Jeweller, Doctor and Engineer.

(2) The doctor is the grandfather of F, who is a Psychologist.

(3) The Manager D is married to A.

(4) C, the Jeweller, is married to the Lawyer.

(5) B is the mother of F and E.


(6) There are two married couples in the family.

(7) The Psychologist is a female while Engineer is a male.



37. What is the profession of E ?

(A) Doctor

(B) Engineer

(C) Manager

(D) Psychologist

Ans : (B)




38. How is A related to E ?

(A) Brother

(B) Uncle

(C) Father

(D) Grandfather

Ans : (D)



39. How many male numbers are there in the family ?

(A) One


(B) Three

(C) Four

(D) Two

Ans : (B)



40. What is the profession of A ?

(A) Doctor

(B) Lawyer

(C) Jeweller


(D) Manager

Ans : (A)



41. Which of the following is one of the pairs of couples in the family ?

(A) AB

(B) AC

(C) AD

(D) AE

Ans : (C)




42. Three of the following four are alike in a certain way and so form a group. Which is the one that does not belong to that group ?

(A) Papaya

(B) Mango

(C) Jackfruit

(D) Watermelon

Ans : (C)



43. Three of the following four are similar in relation to their positions in the English alphabet and hence form a group. Which one does not belong to that group ?

(A) SPEAK : PZVKH


(B) HUSKY : BPGFS

(C) BRAIN : MRZIY

(D) BREAK : PZVIY

Ans : (B)



44. Three of the following four groups of letters are alike in some way while one is different. Find out which one is different ?

(A) DJWR

(B) EKXR

(C) KQDX


(D) AGTN

Ans : (A)



45. Four groups of letters are given below. Three of them are alike in a certain way while one is different. Choose the odd one.

(A) GWOURV

(B) LZKMSU

(C) SFXPMG

(D) JOEHNP

Ans : (C)




46. Four pairs of words are given below out of which the words in all pairs except one, bear a certain common relationship. Choose the pair in which the words are differently related.

(A) Watt : Power

(B) Ampere : Current

(C) Pascal : Pressure

(D) Radian : Degree

Ans : (D)




47. Number of letters skipped in between adjacent letters of the series starting from behind are increased by one. Which of the following series observes this rule ?

(A) ONLKI

(B) OMKIG

(C) OMJFA

(D) OIGDC

Ans : (C)



48. If the letters of the word ‘PROTECTION’ which are at odd numbered position in the English alphabet are picked up and are arranged in alphabetical order from left and if they are now substituted by Z, Y, X and so on, beginning from left which letter will get substituted by X ?


(A) E

(B) O

(C) T

(D) I

Ans : (D)



49. How many pairs of letters are there in the word OPERATION in which the difference between them is the same as in the English alphabet ?

(A) 3

(B) 5


(C) 7

(D) 9

Ans : (C)



50. Arrange the given words in order in which they occur in the dictionary and then choose the correct sequence.

(1) Dissipate

(2) Dissuade

(3) Disseminate

(4) Distract


(5) Dissociate

(6) Dissect

(A) 6, 3, 1, 5, 2, 4

(B) 1, 6, 3, 2, 4, 5

(C) 3, 6, 1, 2, 5, 4

(D) 4, 6, 3, 1, 5, 2

Ans : (A)



51. The letters skipped in between the adjacent letters in the series are followed by equal space. Which of the following series observes this rule ?


(A) SUXADF

(B) RVZDFG

(C) HKNGSW

(D) RVZDHL

Ans : (D)



Directions—(Q. 52–56) In a certain code, letters of English alphabet are coded as given for some words. The numeric code for each letter is given in bracket under coded form and corresponds to the letter in the word in the same serial order, study the coded forms of the given words and find out the rules for their classification. Applying those rules find out the code for the words given in capital letters in the questions that follow—

Word Codes FormATE (0) (5) (0)


NONE (5) (25) (5) (25)

UNIT (30) (5) (30) (5)

PIN (5) (10) (5)

PAGE (5) (25) (5) (25)

OPEN (30) (5) (30) (5)

ONE (0) (5) (0)

CUT (5) (10) (5)

SEAT (5) (15) (15) (5)

DEEP (5) (20) (20) (5)




52. VINA

(A) (5) (0) (5) (15)

(B) (5) (25) (5) (25)

(C) (5) (30) (5) (30)

(D) (5) (10) (5) (30)

Ans : (B)



53. AGE

(A) (0) (15) (0)


(B) (15) (15) (15)

(C) (0) (10) (10)

(D) (0) (5) (0)

Ans : (D)



54. PEAR

(A) (5) (15) (15) (5)

(B) (5) (25) (5) (25)

(C) (5) (10) (5) (10)


(D) (5) (30) (5) (30)

Ans : (A)



55. TIN

(A) (0) (5) (0)

(B) (5) (0) (5)

(C) (0) (10) (0)

(D) (5) (10) (5)

Ans : (D)




56. UNIT

(A) (5) (30) (5) (30)

(B) (5) (10) (30) (10)

(C) (30) (5) (30) (5)

(D) (15) (10) (10) (15)

Ans : (C)



57. If the first and second digits in the squence 7 9 8 4 5 3 6 7 8 3 4 5 are interchanged, also the third and fourth digits, the fifth and sixth digits and so on which digit would be the sixth counting from your right ?

(A) 5


(B) 6

(C) 7

(D) 8

Ans : (C)



58. The letter I, J, K, L, M, N, O, P, Q, R, S, T in their order are substituted by twelve numbers but not in that oder. 3 is assigned to R. The difference between R and M is 7. The difference between K and M is 2. What number is assigned to K ?

(A) 8

(B) 12

(C) 7


(D) 11

Ans : (B)



59. Below are given six three-character numbers. The characters comprise of digits and letters. The letter stands for one less than its serial order in the English alphabet. What will be the middle character of the 3rd number when the numbers are arranged in the decending order ?

8G6, 3DJ, F4C, 7HB, 4E6, B8I

(A) 7

(B) 8

(C) 6

(D) 4


Ans : (D)



60. Which of the following will be the third digit of the fourth number after the following numbers are arranged in descending order after reversing the positions of the digits within each number ?

645, 869, 458, 347, 981, 792

(A) 4

(B) 6

(C) 8

(D) 9

Ans : (A)

********************************************
(Held on 9-5-2010) 


1. Which of the following stars lies nearest to our solar system ?

(A) Barnard’s Star


(B) Sirius A

(C) Alpha Centauri A

(D) Proxima Centauri

Ans : (D)



2. Of the total water on the earth, fresh water reserves constitute approximately—

(A) 5•8%

(B) 4•5%


(C) 2•59%

(D) 1•2%

Ans : (C)



3. The term ‘Bishop’ in sports is related to—

(A) Chess

(B) Hockey


(C) Badminton

(D) Bridge

Ans : (A)



4. Yakshagan is the famous dance form of the State of—

(A) Orissa

(B) Andhra Pradesh

(C) Tamil Nadu

(D) Karnataka


Ans : (D)



5. Balance of Payments means—

(A) Difference between export and imports

(B) Balance to be paid to the exporters

(C) Balance to be paid to the industrialists

(D) Balance coming in allocation of funds for States

Ans : (A)






6. A light year is equivalent to about—

(A) 365 days

(B) Six million miles

(C) Six billion miles

(D) Six trillion miles

Ans : (D)



7. Denmark, Iceland, Norway, Sweden and Finland are together called—

(A) Slovakia


(B) Scandinavia

(C) Netherlands

(D) Australasia

Ans : (B)



8. The device used for locating submerged objects under sea is—

(A) Sonar

(B) Radar

(C) Laser


(D) Maser

Ans : (A)



9. Which article of the Constitution provides the Parliament the power to ammend Constitution ?

(A) 376

(B) 370

(C) 368

(D) 390

Ans : (C)




10. What is the name of the first successfully cloned deer ?

(A) Dawn

(B) Deluxe

(C) Demor

(D) Dewey

Ans : (D)



11. The idea of motion pictures was propounded by—

(A) N. R. Finsen


(B) T. A. Edison

(C) J. L. Baird

(D) Berliner

Ans : (B)



12. In Greek mythology, Apollo is the god of which of the following ?

(A) Love

(B) Peace

(C) Prophecy


(D) Medicine

Ans : (C)



13. NASA’s new space telescope is—

(A) Wise

(B) Rise

(C) Barack

(D) Telle

Ans : (A)




14. Bull fighting is the national game of—

(A) Italy

(B) Poland

(C) Spain

(D) Sudan

Ans : (C)



15. In a rare recognition, the Government of Mauritius decides to introduce a book written by Uttarakhand Chief Minister Ramesh Pokhriyal Nishank in the School Syllabus of the island nation, what is the name of the book ?

(A) Sound of Hills


(B) Sparsh Ganga

(C) Heaven

(D) Amrit

Ans : (B)



16. Who has been selected for the ‘Niwano Peace Prize, 2010’ for her contribution to the uplift of poor women in India ?

(A) Ela Bhatt

(B) Jhumpa Lahiri


(C) Punam Suri

(D) Mandakni Apte

Ans : (A)



17. On which of the following dates in 2010 did the safeguard agreement with regard to civilian nuclear facilities between India and the International Atomic Energy Agency (IAEA) come into force ?

(A) February 28

(B) February 21

(C) February 11

(D) February 4


Ans : (D)



18. The tax which the kings used to collect from the people in the Vedic period was called—

(A) Bali

(B) Vidatha

(C) Varman

(D) Kara

Ans : (B)



19. The cells which are responsible for the production of antibodies are—


(A) Red blood cells

(B) Neutrophils

(C) Lymphocytes

(D) Platelets

Ans : (C)



20. Correct expansion of the term ‘http’ in Internet address is—

(A) Hybrid text transfer protocol


(B) Hyper text transfer protocol

(C) Higher transfer text protocol

(D) Higher text transfer protocol

Ans : (B)
***************************************************
(Held on 6-6-2010)
1. Jawaharlal Nehru National Solar Mission or Solar India has been launched to create an installed capacity of 2000 MW by the end of—

(A) 13th Plan

(B) 14th Plan

(C) 15th Plan

(D) 16th Plan


Ans : (A)



2. Bollywood actors who, on February 2, 2010 rang bells in NASDAQ on Time Square, New York—an honour usually reserved for CEOs of major corporations, are—

(A) Amir Khan and Priyanka Chopra

(B) Shah Rukh Khan and Kajol

(C) Abhishek Bachchan and Aswarya

(D) Saif Ali Khan and Kareena Kapoor

Ans : (B)




3. The term ‘16 yards hit’ is associated with—

(A) Badminton

(B) Baseball

(C) Polo

(D) Hockey

Ans : (D)



4. The vital constituent of blood that helps in clotting is—


(A) Platelets

(B) Haemoglobin

(C) Plasma

(D) Serum

Ans : (A)



5. Which of the following animals can hear ultrasonic sound ?

(A) Cat

(B) Rat


(C) Bat

(D) Squirrel

Ans : (C)





6. The acid rain destroys vegetations because it contains—

(A) Sulphuric acid

(B) Ozone

(C) Carbon monoxide

(D) Nitric acid


Ans : (A)



7. Man Booker Prize winner Ms. Kiran Desai’s novel, ‘The Inheritance of Loss’ deals with—

(A) Telangana movement

(B) Santhal movement

(C) Gorkhaland movement

(D) Bodoland movement


Ans : (C)



8. GARC is the acronym for—

(A) Global Automotive Research Consortium

(B) Global Atomic Research Consortium

(C) Global Automotive Research Centre

(D) Global Advancement Research Centre

Ans : (C)



9. The largest fresh water lake, ‘Lake Superior’ is located in—


(A) Canada

(B) Russia

(C) Tanzania

(D) Argentina

Ans : (A)



10. In which State of India is the largest river island Majuli ?

(A) Assam

(B) Kolkata


(C) Orissa

(D) M.P.

Ans : (A)



11. Match the following—

(a) Berlin, Germany

(b) Ankara, Turkey

(c) Bristol, UK

(d) Budapest, Hungary


1. Danube

2. Kizil

3. Spree

4. Avon

Codes :

(a) (b) (c) (d)

(A) 2 1 4 3

(B) 4 3 1 2

(C) 1 4 2 3


(D) 3 2 4 1

Ans : (D)



12. Government’s ambitious UID Project has been renamed—

(A) Parichaya

(B) Pahchan

(C) Aadhar

(D) Aanklan

Ans : (C)




13. Who is the author of the book ‘Speaking for Myself’ ?

(A) Salman Rushdie

(B) Cherie Blair

(C) Mohammad Hanif

(D) Hillary Clinton

Ans : (B)




14. The name of India’s first homebuilt stealth warship whose features reduce the probability of being detected at sea is—

(A) Sahyadiri

(B) Satpura

(C) Shivalik

(D) Shivaji

Ans : (C)



15. Which country will host the 17th SAARC summit ?

(A) Bangladesh


(B) Nepal

(C) Shri Lanka

(D) Maldives

Ans : (D)



16. India’s Prime Minister Manmohan Singh is seated at the ……… spot in the leaders’ list of 100 most influential persons of the world realeased by the famous Time Magazine recently.

(A) 15


(B) 19

(C) 23

(D) 27

Ans : (B)



17. The Islam was established in—

(A) 5th A.D.

(B) 7th A.D.

(C) 3rd B.C.


(D) 5th B.C.

Ans : (B)



18. Which part of the human body is highly affected by the nuclear radiation first ?

(A) Bone marrow

(B) Skin

(C) Lungs

(D) Eyes

Ans : (B)




19. Who invented the video-tape ?

(A) Charles Ginsberg

(B) Georges de Mestral

(C) Richard James

(D) P. T. Farnsworth

Ans : (A)



20. Match the following—

(a) NH–1


(b) NH–3

(c) NH–5

(d) NH–7

1. Varanasi–Kanyakumari

2. Kolkata–Chennai

3. New Delhi–Amritsar


4. Agra–Mumbai

Codes :

(a) (b) (c) (d)

(A) 2 1 4 3

(B) 3 4 2 1

(C) 4 3 1 2

(D) 1 3 4 2

Ans : (B)

******************************************

1. Which of the following is a General Insurance Company functioning in India ?


(A) Life Insurance Corporation of India

(B) ICICI Prudential

(C) Tata AIG

(D) United India Insurance Company

(E) All are General Insurance Companies

Ans : (D)



2. Who amongst the following is the Regulator of Insurance sector in India ?

(A) RBI


(B) AMFI

(C) IRDA

(D) SEBI

(E) All of these

Ans : (C)



3. Which of the following terms is NOT used in insurance sector ?

(A) Indemnity

(B) Coverage


(C) Misuse Alert

(D) Casualty

(E) Annuity

Ans : (C)





4. Which of the following Public Sector companies/organizations provides insurance cover to exporters ?

(A) ECGC

(B) NABARD


(C) SIDBI

(D) IRDA

(E) None of these

Ans : (A)



5. The main feature of the National Agricultural Insurance Scheme is to insure which of the following?

(A) Life of the farmer

(B) Crop of the farmer

(C) Animals who are used in agricultural activities


(D) Land of the farmer

(E) Both Land and family of the farmer

Ans : (B)



6. In which one of the following States was the first UID card issued ?

(A) Gujarat

(B) Manipur

(C) Assam

(D) West Bengal


(E) Maharashtra

Ans : (E)



7. Which of the following books is written by Anita Desai ?

(A) Magic Seeds

(B) The Village by the Sea

(C) A Bend in the River

(D) My God Died Young

(E) None of these


Ans : (B)



8. Expand the term MGNREGA—

(A) Mahatma Gandhi National Rural Employment Guarantee Act

(B) Mahatma Gandhi National Rural Employment Guarantee Agency

(C) Mahatma Gandhi National Rural Employment Generation Act

(D) Mahatma Gandhi National Rural Employment Generation Agency

(E) None of these

Ans : (A)




9. Which of the following terms is used in the field of finance and banking?

(A) Elasticity

(B) Inflation

(C) Pulse Rate

(D) Hot waves

(E) Plasma

Ans : (B)



10. With which one of the following sports, Saina Nehwal is associated ?


(A) Chess

(B) Badminton

(C) Long Jump

(D) Lawn Tennis

(E) Table Tennis

Ans : (B)



11. Which one of the following States has passed a bill to regulate the interest rates on micro-finance ?

(A) Odisha


(B) Haryana

(C) Karnataka

(D) Kerala

(E) Andhra Pradesh

Ans : (E)



12. Which of the following rates are reviewed by the RBI at the time of periodical review of the policy ?

(a) Bank Rate

(b) Repo Rate


(c) Savings Bank Rate

(A) Only (a)

(B) Only (b)

(C) Both (a) and (b)

(D) All (a), (b) and (c)

(E) Only (c)

Ans : (D)



13. Which one of the following is not included in TAPI gas project ?


(A) Turkmenistan

(B) Pakistan

(C) Afghanistan

(D) India

(E) Iran

Ans : (E)



14. Which one of the following States has passed a legislation to confiscate the property of corrupt officials and to open schools in these properties ?

(A) West Bengal


(B) Jharkhand

(C) Bihar

(D) Rajasthan

(E) Haryana

Ans : (C)



15. Brihadeeswara Temple has recently celebrated its millennium birthday. In which of the following States is it located ?

(A) Karnataka

(B) Andhra Pradesh


(C) Kerala

(D) Odisha

(E) Tamil Nadu

Ans : (E)



16. In which one of the following States, Jaitapur Nuclear Power plant is proposed to be set up ?

(A) Gujarat

(B) Haryana

(C) Odisha


(D) Karnataka

(E) Maharashtra

Ans : (E)



17. According to Planning Commission, in how many years shall the per capita income in India become double ?

(A) Nine

(B) Five

(C) Seven

(D) Three


(E) None of these

Ans : (A)



18. Expand the term IFRS.

(A) Indian Financial Reporting Standards

(B) Indian Financial Reporting Systems

(C) International Financial Reporting Standards

(D) International Financial Reporting Systems

(E) None of these


Ans : (C)



19. Who is Liu Xiaobo ?

(A) North Korean dissident, who has been awarded Nobel peace prize

(B) Chinese citizen, who has been awarded Nobel peace prize

(C) North Korean dissident, who has been awarded Nobel literature prize

(D) Sportsman who was awarded maximum 8 Gold medals in Commonwealth

(E) None of these

Ans : (B)




20. Who is Hardeep Puri ?

(A) India’s permanent representative at UN

(B) Vice President, IMF

(C) Vice President, World Bank

(D) India’s ambassador in USA

(E) India’s ambassador in UK

Ans : (A)




21. With which one of the following games is Ishant Sharma associated ?

(A) Chess

(B) Badminton

(C) Table Tennis

(D) Volley Ball

(E) Cricket

Ans : (E)



22. With which one of the following games, France’s Zinedine Zidane is associated ?


(A) Hockey

(B) Football

(C) Snooker

(D) Volley Ball

(E) Base Ball

Ans : (B)



23. As per recent newspaper reports, which one of the following countries was second largest importer of goods and services in the world during year 2009-10 ?

(A) China


(B) USA

(C) Japan

(D) Russia

(E) India

Ans : (E)



24. According to RBI governor, which one of the following is the challenge in its monetary policy ?

(A) Inflation

(B) Inflation and Supportive recovery


(C) Inflation and fiscal deficit

(D) Inflation and high cost of imports

(E) None of these

Ans : (C)



25. Which one of the following Public Sector Organisations has offered largest ever equity offer ?

(A) SAIL

(B) Coal India

(C) Power Grid


(D) Shipping Corporation of India

(E) None of these

Ans : (B)



26. Through which one of the following sources domestic funds are raised by Companies ?

(a) IPO only

(b) FPO only

(c) Commercial papers

(A) Only (a) and (b)


(B) All (a), (b) and (c)

(C) Only (a) and (c)

(D) Only (a)

(E) Only (c)

Ans : (C)



27. Which one of the following was the reason owing to which Govt. want Reserve Bank of India to tighten prudential norms for NBFCs ?

(A) To reduce liquidity in the market

(B) It is as per Basel II requirements


(C) It is as per Bank for International Settlement (BIS) directives

(D) It is to protect NBFCs from any impact of possible economic slowdown

(E) None of these

Ans : (D)



28. In the financial year 2009-10, which one of the following Banks has made highest Total dividend Payout ?

(A) SBI

(B) PNB

(C) Bank of India


(D) Canara Bank

(E) None of these

Ans : (E)



29. In respect of which one of the following countries, India has proposed non payments for import of crude oil ?

(A) Iran

(B) Iraq

(C) Kuwait

(D) Sudan


(E) U.A.E.

Ans : (A)



30. Through which one of the following methods, RBI has allowed Indian companies to hedge exchange rate risks associated with trade transactions ?

(A) Forwards

(B) FRA

(C) Swaps

(D) Currency options

(E) Cross currency options


Ans : (D)



31. Which of the following books is written by V. S. Naipaul ?

(A) A Handful of Dust

(B) A House for Mr. Biswas

(C) A Passage to India

(D) Darkness at Noon

(E) None of these

Ans : (B)




32. With which one of the following countries, India has signed agreement to build Multi Model Transit Transport Project ?

(A) Bangladesh

(B) Myanmar

(C) Sri Lanka

(D) Nepal

(E) Maldives

Ans : (B)



33. Which one of the following countries is Number One Country in terms of Purchasing Power ?


(A) USA

(B) Germany

(C) China

(D) South Korea

(E) None of these

Ans : (A)



34. Which one of the following directions has been given by Government to Civic bodies ?

(A) They must use 70 per cent of the funds for BPL families


(B) They must use 70 per cent of the funds for the poor living within their jurisdiction

(C) They must use 25 per cent of the funds for BPL families

(D) They must use 25 per cent of the funds for the poor living within their jurisdiction

(E) None of these

Ans : (B)



35. On which one of the following issues a group of business leaders and other eminent citizens have expressed concern ?

(A) E-Governance

(B) Governance


(C) Fiscal deficit

(D) Governance deficit

(E) None of these

Ans : (B)



36. ‘Yuan’ is the currency of which one of the following countries ?

(A) Japan

(B) South Korea


(C) North Korea

(D) Taiwan

(E) China

Ans : (E)



37. Which one of the following is a leading power in 17 nation Euro Region ?

(A) Germany

(B) France

(C) Norway


(D) Greece

(E) Portugal

Ans : (A)



38. What are teaser loan rates charged by banks ?

(A) Fixed rate of interest charged by banks

(B) Floating rate of interest charged by banks

(C) Rate of interest in the initial period is less and goes up subsequently

(D) Rate of interest in the initial period is more and it goes down subsequently


(E) None of these

Ans : (C)



39. Expand the term FSDC which is used in financial sectors ?

(A) Financial Security and Development Council

(B) Financial Stability and Development Council

(C) Fiscal Security and Development Council

(D) Fiscal Stability and Development Council

(E) None of these


Ans : (B)



40. According to the 8th Annual Global Retail Development Index (GRDI), which one of the following countries is ‘most attractive’ retail market in the world ?

(A) China

(B) India

(C) UAE

(D) Saudi Arabia

(E) Japan


Ans : (B)



41. Govt.’s ‘Food-for-work’ programme means—

(A) to pay in kind to work and build rural infrastructure

(B) supplying balanced diet to workers in rural areas

(C) no-work-no-pay principle

(D) ensuring enough food to rural worker-households


(E) None of these

Ans : (E)



42. The insurance companies collect a fixed amount from its customers at a fixed interval of time. What

is it called ?

(A) Instalment

(B) Contribution

(C) Premium

(D) EMI


(E) Service Charge

Ans : (A)



43. Which of the following is/are the various types of insurance ?

(a) Life Insurance

(b) Health Insurance

(c) Liability Insurance

(A) Only (a)

(B) Only (b)


(C) Only (c)

(D) All (a), (b) and (c)

(E) Only (a) and (b)

Ans : (D)



44. What is the full form of ‘ESOPs’ as seen in financial newspapers ?

(A) Employee Stock Ownership Plan

(B) Executive Salary Options


(C) Emergency Stock Operations

(D) Early Sales Opportunities

(E) Executive Stock Ownership

Ans : (A)



45. Who are considered Super Senior Citizen as per budget 2011-12 ? Those who have completed a minimum age of—

(A) 65 years

(B) 70 years

(C) 75 years


(D) 80 years

(E) 90 years

Ans : (D)



46. Pohang Steel Company (POSCO) is a company originally based in—

(A) China

(B) Vietnam

(C) USA

(D) Italy


(E) South Korea

Ans : (E)



47. As per the revised data released by the Central Statistical Organisation, the GDP growth in 2010 has been at which of the following levels ?

(A) 6%

(B) 6•5%

(C) 8%

(D) 7•5%


(E) None of these

Ans : (C)



48. Who amongst the following was awarded Padma Vibhushan recently (2011) ?

(A) Kumar Mangalam Birla

(B) Harbhajan Singh

(C) Nirupama Rao

(D) Montek Singh Ahluwalia

(E) Saina Nehwal


Ans : (D)



49. Annual meeting of World Economic Forum was organized in January 2011 in which of the following cities ?

(A) Davos

(B) Milan

(C) Tokyo

(D) London

(E) Lisbon

Ans : (A)




50. India recently signed a deal on the sharing of Teesta and Feni river waters. This agreement is signed with which of the following countries ?

(A) Nepal

(B) China

(C) Pakistan

(D) Bangladesh

(E) Bhutan

Ans : (D)

*************************************************
1. What is Siam now known as ?

(A) Laos

(B) Thailand

(C) Vietnam


(D) Cambodia

Ans : (B)



2. The correct group of cities, through which National Highway No. 8 passes is—

(A) Bikaner, Ajmer, Kota

(B) Jodhpur, Jaipur, Bikaner

(C) Jaipur, Udaipur, Ahmedabad

(D) Jaipur, Pune, Udaipur

Ans : (C)




3. Through its power of judicial review, the Supreme Court ensures—

(A) Judicial Supremacy

(B) A democrtic government in the country

(C) A constitutional government in the country

(D) That Parliament Sovereignty is curbed

Ans : (D)





4. Which unit of valuation is known as ‘Paper Gold’ ?


(A) Petrodollar

(B) SDR

(C) Eurodollar

(D) GDR

Ans : (B)



5. What is the playing time of the full version of the Indian National Anthem ?

(A) 47 seconds

(B) 50 seconds


(C) 52 seconds

(D) 60 seconds

Ans : (C)



6. When was Barack Obama sworn in as the 44th President of U.S.A. ?

(A) January 15, 2009

(B) January 20, 2009

(C) January 25, 2009

(D) January 30, 2009


Ans : (B)



7. The Movie ‘Slumdog Millionaire’ is based on a novel ‘Q and A’ written by an Indian—

(A) Khuswant Singh

(B) Khalid Mohammad

(C) Sabarjit Singh


(D) Vikas Swarup

Ans : (D)



8. Who has been appointed the first Director-General of the ‘National Investigation Agency (NIA)’ ?

(A) P. S. Sharma

(B) A. Gopalswami

(C) Radha Vinod Raju

(D) K. C. Verma


Ans : (C)



9. Under which Article of the Indian Constitution, can the President of India withhold his assent to a Bill passed by both the houses of Parliament ?

(A) Article, 100

(B) Article, 111

(C) Article, 200

(D) Article, 222

Ans : (B)



10. Which language is spoken by the people of Tripura ?


(A) Bengali

(B) Telugu

(C) Assamese

(D) Malayalam

Ans : (A)



11. Surya Shekhar Ganguly is a player of—

(A) Chess

(B) Cricket


(C) Golf

(D) Hockey

Ans : (A)



12. We very frequently read about Special Economic Zones (SEZs) in newspapers. These SEZs were established with which of the following objectives ?

1. To protect domestic market from direct competition from multinationals

2. To provide more capital to agricultural and allied activities

3. To attract foreign investment directly

(A) Only 1


(B) Only 2

(C) Only 3

(D) 1, 2 and 3 all

Ans : (C)



13. With which is ‘Uro Cup’ associated ?

(A) Cricket

(B) Football


(C) Polo

(D) Wrestling

Ans : (B)



14. World Environment Day is on—

(A) June, 5

(B) June, 11

(C) June, 17

(D) June, 30


Ans : (A)



15. Which rock-shelter in India bears largest number of paintings ?

(A) Adamgarh

(B) Bhimbetka

(C) Ghagharia

(D) Lekhia

Ans : (B)



16. Heat received by earth from the sun is known as—


(A) Isolation

(B) Solar heat

(C) Solar radiation

(D) Thermal radiation

Ans : (A)



17. Who invented Coronograph which allowed the study of the sun without waiting for a total solar eclipse ?

(A) Donald Menzel

(B) Pierre Jenseen


(C) John Evershed

(D) Bernard Lyot

Ans : (D)



18. Who wrote ‘The Structure of the Universe’, giving in detail the origin of the universe and its present status, among other things ?

(A) Robert Jastrow

(B) S. Chandrashekhar

(C) Jayant V. Narlikar


(D) Carl Sagan

Ans : (C)



19. Name the Sri Lankan town which has been the Tamil Tiger’s administrative and political headquarters and has been captured by the army recently ?

(A) Kandy

(B) Jaffana

(C) Kilinochchi

(D) Mullaithivu

Ans : (C)




20. Which committee recommended abolition of tax rebates under Section, 88 ?

(A) Kelkar Committee

(B) Rangarajan Committee

(C) Chelliah Committee

(D) Shome Committee

Ans : (D)
*******************************

Reasoning Ability

Directions—(Q. 1—5) In the following questions, the symbols +, -,x


÷ and = are used with the following meanings

A + B means A is greater than B;

A – B means A is greater than or equal to B;

A x B means A is equal to B;

A ÷ B means A is smaller than B; and

A = B means A is smaller than or equal to B.


For each question you have to assume given statements to be true and then decide which of the two given conclusions is are definitely true. Give answer

(A) If only conclusion I is true;

(B) If only conclusion II is true;

(C) If either conclusion I or conclusion II is true; and

(D) If neither conclusion I nor conclusion II is true.

1. Statements:

D=S, N + V, D x R, R + V

Conclusions: I. S + V

II D+N




2. Statements:

K x P, M + J, C÷ P, K = M

Conclusions: I. M + P

II. M x P

3. Statements:

L – R, N x M, L ÷ M, N = P


Conclusions: I. M + R

II. R+P

4.Statements:

W + K, S = Z, X – W, S x K

Conclusions: I. K x Z

II. X÷K

5. Statements:


P ÷ Q, W – N, P — N, T + W

Conclusions: I. P x W

II. Q÷W

Directions—(Q. 6—10) There are two or three statements followed by four conclusions numbered I, II , III and IV. Take the given statements to be true even if they seem to be at variance with commonly known facts and then decide which of the given conclusions logically follow(s) .


6. Statements:

Some songs are throngs.

Some throngs are longs.

Conclusions:

I. Some songs are longs.

II. No songs are longs.

III. Only longs are songs.

IV Only songs are longs.

(A) Only I Follows


(B) Only III follows

(C) Either I or II follows

(D) Either III or IV follows

7. Statements:

Some charts are darts.

All darts are carts.

Some carts are smarts.

Conclusions:

I. Some charts are carts.


II. Some carts are darts.

Ill. Some darts are smarts.

IV. Some smarts are charts.

(A) Only I and II follow

(B) Only I and III follow

(C) Only II and III follow

(D) Only I, III and IV follow

8. Statements:

All blanks are beams.


All bridges are beams.

All bridges are cows.

Conclusions:

I. Some cows are beams.

II. Some blanks are cows.

III. Some blanks are bridges.

IV. Some bridges are not cows.

(A) Only I follows

(B) Only II follows


(C) I,II and III follow

(D) None follow



9. Statements:

Some Chinese are not Russians.

All Russians are Africans.

Some Africans are Indians.

Conclusions:

I. Some Chinese are not Africans.


II. Some Chinese are not Indians.

Ill. All Russians are Indians.

IV. Some Indians are Chinese.

(A) Only II follows

(B) Only III follows

(C) Only IV follows

(D) Either II or IV follows

10. Statements:

All chalks are cheese.


No cheese are ships.

Some herds are ships.

Conclusions:

I. Some herds are not chalks.

II. Some herds are cheese.

Ill. Some cheese are not herds.

IV. No chalk is a ship.

(A) I, II and IV follow

(B) Either II or III and IV follows


(C) II and III follow

(D) II and IV follow

Directions—(Q. 11—15) A word arrangement machine, when given a particular input, rearranges it following a particular logic. The following is the illustration of the input and the steps of arrangement?

Input: CRI END YAM STU THE

StepI:YAM THE CRI END STE

StepII:YAM THE STE END CRI

Step III:STU CRI YAM THE END

Step IV;STU CRI END THE YAM


Study the logic and answer questions

11. If step VII of an input is ‘OVER THE PRE NEW BONE’ what is

the step IV of that input?

(A) BONE THE PRE OVERNEW

(B) THE PRE BONE NEW OVER

(C) THE BONE PRE OVER NEW

(D) PRE BONE THE OVER NEW


12. Given the following input— SYM REACH LAD PHOTO CAL

what step will the following arrangement?

REACH LAD PHOTO SYM CAL

(A) VI

(B) V

(C) IV

(D) III

13. If Step VI of a given input be ‘MAP IND PAK RUS ENG’, what would be the input?


(A) IND MAP PAK RUS ENG

(B) IND MAP ENG RUS PAK

(C) ENG RUS IND PAK MAP

(D) RUS MAP IND ENG PAK

14. Given the following input, what would be Step VIII of the input?

Input : BANK CRIS ATTRACT WITH PRIZE

(A) PRIZE WITH CRIS ATTRACT BANK

(B) CRIS BANK PRIZE WITH ATTRACT

(C) PRIZE WITH BANK ATTRACT CRIS


(D) CR15 BANK ATTRACT WITH PRIZE

15. In which step will we get the same arrangement as the input?

(A) VIII

(B) IX

(C)X

(D)XI

Directions—(Q. 16—20) In each question below is given a statement followed by three assumptions numbered I II and III. You have to consider the statement and the assumption, and decide which of the assumptions is implicit in the statement.

Then decide which one of the answers, (A), (B), (C) and (D) is correct.


16. Statement : Considering the tickets sold during the last seven days, the circus authorities decided to continue the show for another fortnight which includes two weekends.

Assumptions:

I. People may not turn up on weekdays.

II. The average number of people who will be visiting circus will be more or less the same as that of the last seven days.

III. There may not be enough responses at other places.

(A) None is implicit

(B) Only II is implicit

(C) Both I and II are implicit

(D) Only III is implicit


17. Statement : The telephone company informed the subscribers through a notification that those who do not pay their bills by the due date will be charged penalty for every defaulting day.

Assumptions:

I. Majority of the people may pay their bills by the due date to avoid penalty.

II. The money collected as penalty may set off the losses due to delayed payment.

III. People generally pay heed to such notices.

(A) Only I and II are implicit

(B) Only II and III are implicit

(C) Only I and III are implicit

(D) All are implicit


18. Statement The national air carrier has decided to start a weekly air service from town ‘A’ to town ‘B’.

Assumptions:

I. There will be enough passengers to make the operation economically viable.

II. Other carriers may not start such service.

III The people staying around these towns can afford the cost of air travel.

(A) Only I is implicit


(B) Both II and III are implicit

(C) Both I and II are implicit

(D) ALL are implicit

19. Statement : A group of friends decided to go for a picnic to Dum Duma Lake during the next holiday season to avoid crowd of people.

Assumptions:

I. Generally many people do not go to Dum Duma Lake.

II. People prefer other spots to Dum Duma Lake.

III. Many people do not know about Dum Duma Lake.

(A) Only I is implicit


(B) Only II is implicit

(C) Only I and II are implicit

(D) All are implicit

20. Statement : “Wanted a two bedroom flat in the court area for immediate possession”—Advertisement.

Assumptions:

I. Flats are available in court

II. Some people will respond to the advertisement.

Ill. It is a practice to give such an advertisement.


(A) Only I and II are implicit

(B) Only II is implicit

(C) Only III is implicit

(D) All are implicit

21. pq—rqp—qrrq—p—rrqpp—r

(A) rqpqr


(B) qprrq

(C) qrprp

(D) rppqq

22. nnt—qqn—tt—qnn—tq-.

(A) nnqtq

(B) qntnq

(C) tnqtq


(D) ttqtq

Directions(Q. 23 and 24)

P + Q means P is the brother of Q

P – Q means P is the sister of Q

P x Q means P is the father of Q

P ÷ Q means P is the mother of

Q.


23. A÷B-C x D,

then A is D

(A) Sister

(B) Aunt

(C) Mother

(D) Grandmother

24. Which of the following shows that V is the grandmother of Y?

(A) VxR+KVY


(B) V÷RxK-Y

(C) V÷R+KxY

(D) VxR-K÷Y

25. How many pairs of letters are there in the word STRAIGHT FORWARD which have numbers of letters between them in the word equal to the number of letters between them in English alphabet?

(A)4

(B)5

(C)6


(D)7

26. Milk: Butter

(A) Banana : Fruit

(B) Juice : Health

(C) Wood: Paper

(D) Chili Spices

27. Hat : Headwear

(A) Shark : Fish

(B) Crocodile : Terrapin


(C) Shoe : Socks

(D) Glove: Hand

28. Horse : Cow

(A) Milk: Ice-cream

(B) Mars : Moon

(C) Snow : Water

(D) Pink: Blue

29. Hair : Head

(A) Teeth : Mouth


(B) Tea-leaves : Mountain slopes

(C) Hand: Arm

(D) Footpath : Road

30. Fore: Hind

(A) Sky:Space

(B) Land:Sea

(C) North : South

(D) Face:Neck

Directions—(Q. 31—35) Read the following information to answer the questions— A, B, C, D and E are sons of P, Q ,R, S and T but not in the same order.


Match the right mother and son on the basis of the information given below—

Q is not B’s or C mother.

A’s or Es mother is not T.

C is not R’s son.

E is not Q’s or S’s son and his mother’s name does not start with the letter ‘R’.


A’s mother is not Q or R

31. Who is S’s son?

(A)A

(B)B

(C)C

(D)D

32. Who is C mother?

(A)P


(B)R

(C)S

(D)T

33. Who is Q’s son?

(A)A

(B)D

(C)B

(D)E


34. Who is P’s son?

(A)B

(B)C

(C)D

(D)E

35. Who is son of R ?

(A)E

(B)A


(C)B

(D)D

Lflrections—(Q. 36-40) In each of the following number series, two terms are put within brackets. Mark in the answer-sheet—

(A) If both the bracketed terms are right;

(B) If the first bracketed term is right and second is wrong;

(C) If the first bracketed term is wrong and second is right;

(D) If both the bracketed terms are wrong.

36. 5, 17, (30), 73, 129, (225)


37. 11, 29, (55), 89, (131), 181

38. 3, 12, 28, 32, (42), (57), 61, 70, 86

39. 2,3,6, (11), 18, (27), 38

40. 4, 14, (18), (24), 31, 36, 38, 48, 53

Directions—(Q. 41-45 )Choose the odd one.

41.

(A) YNHIA

(B) SGRFI


(C) ISEPU

(D) FHUJU

42.

(A) PROUD

(B) DRIVER

(C) WHEAT

(D) TRAIN

43.

(A) CXHIA


(B) RCFCL

(C) MTOWF

(D) CPRSV

44.

(A) JOT

(B) OUT

(C) TEN

(D)DIN

45. (A) HR


(B) GT

(C) KP

(D) FU

Directions—(Q. 46-49) Read the following information and answer the questions given below it. If ranks of five candidates P, Q ,R S and T are arranged in ascending order of their marks in Numerical Ability, T is the fourth and S is the first. When they are arranged in the ascending order of marks in General Awareness, P takes the place of T and T takes the place of Q. R’s position remains the same in both the arrangements. Q’s marks are lowest in one test and highest in the other test. P has more marks than R in Numerical Ability.

46. Who has secured the highest marks in General Awareness?

(A)Q

(B)T


(C)S

(D)R

47. Who has secured the highest marks in Numerical Ability?

(A)S

(B)R

(C)P

(D)Q

48. Which of the following groups of candidates has improvement in rank in General Awareness as compared to that in Numerical Ability?

(A) SPT


(B) PSR

(C) QST

(D) SPQ

49. Whose marks in General Awareness are more than R’s marks in General Awareness?

(A) Only P’s

(B)P’s, Q’s and S’s


(C) P’s, S’s and T’s

(D) P’s, Q’s and T’s

50. In a certain code CAMEL is written as XPOGT and RABBITS as YPUULFZ. How can AIRMAIL be written in that code?

(A) PIYOPLT

(B) PLYOPTL

(C) PLROPLT


(D) PLYOPLT

Answers:

1 A
2 C
3 A
4 B
5 D
6 C
7 A
8 D
9 D
10 B
11 C
12 B
13 D
14 C
15 C
16 B
17 D
18 C
19 D
20 A
21 D
22 C
23 D
24 B
25 C
26 C
27 D
28 B
29 A
30 C
31 A
32 D
33 B
34 D
35 C
36 C
37 A
38 C
39 A
40 D
41 D
42 B
43 C
44 B
45 A
46 B
47 D
48 A
49 C
50 D
******************************************
1. Which of the following is not the

name of an Insurance Scheme

launched by the Government of

India


(A) ]anashree Bima Yojana

(B) Krishi Shramik Sarnajik

Suraksha Yojana

(C) Shiksha Sahyog Yojana

(D) Varsha Bima Yojana

(E) National Saving Scheme

Programme

Answer.D

2. Which of the following insurance


companies writes its punch

line in the advertisements

”Insurance is the subject matter

of solicitation” ?

(A) CIC

(B) LIC

(C) ING Vysya Life Insurance

Co.


(D) Tata AIG Life Insurance Co.

(E) None of these

Answer.B

3. How much Foreign Institutional

Investment is allowed in

insurance sector at present

(A) 20%

(B) 46%

(C) 59%


(D) 63%

(E) None of these

Answer.E

4. How many public sector companies in India are in Insurance

Business

(A) Only one

(B) Two

(C) Three

(D) Four


(E) More than ten

Answer.A

5. Which of the following was the

parent company of New India

Assurance ?

(A) LIC

(B) GIC

(C) Oriental Insurance Co. Ltd.

(D) United India Insurance


(E) None of these

Answer.B

6. Which of the following organizations of the workers of unorganized sector has adopted an insurance scheme for all its workers and has become a model organization for the workers of the unorganized sector ?

(A) SERC

(B) SEWA

(C) VSNL

(D) SEBI

(E) MTNL

Answer.B


7. Which of the following projects

of the ISRO was recently insured

by the New India Assurance

Company ?

(A) MATSAT-II

(B) Aryabhatt

(C) PSLV

(D) Chandrayaan

(E) lNSAT—2E


Answer.C

8. Which of t.he following industrial

groups of India is Not in any

type of Insurance Business in

India 7

(A) Tatas

(B) Birlas

(C)‘ Iaipurias


(D) Mahindras

(E) All are in Insurance Business

Answer.C

9. Which of the following words/

terms is closely associated with

the insurance business

(A) Archives

(B) Donation

(C) Actuary


(D) Quest

(E) All are associated with

insurance

Answer.C

10. Which of the following is the

abbreviated name of the agency /

organization which is the regulator of insurance business in

India ?

(A) NHB


(B) IRDA

(C) IBA

(D) IDFC

(E) ICRISAT

Answer.B

ll. Insurance services are normally

not provided for which of the

following?

(A) Health


(B) Life

(C) jobs

(D) House & Property

(E) It is available for all

Answer.C

12. The New India Assurance Company was established in 1919

by

(A) Dorab Tata


(B) G. D. Birla

(C) Iamunalal Baja]

(D) Kamlapat Singhania

(E) None of these

Answer.A

13. Which of the following is largest

Non Life Insurance Company in

India ?

(A) ICICI Lombard General


Insurance Company Ltd.

(B) United India Insurance

Company Ltd.

(C) General Insurance Company

Ltd.

(D). New India Assurance Company Ltd.

(E) None of these

Answer.D

14. Ramesh Pokhriyal has taken over


as the new-

(A) Chief Minister of Chhattis-

garh

(B) Chief Minister of Uttara-

khand

(C) Governor of Haryana

(D) Governor of Uttarakhand

(E) Governor of Chhattisgarh

Answer.B


15. As per the announcement made

by the Govt. of India, which of

the following organizations is

set up by it for a very specific

purpose of providing a personal

identity card to all its citizens ?

(A) National Remote Sensing

Agency

(B) Unique Identification


Authority of India (UIA)

(C) National Spot Exchange

(D) Trade Development Authority

(E) People’s Union for Civil

Liberties

Answer.B

16. Which of the following Insurance

Companies was launched with


NABARD as one of its promoter”s with 30% stake in it ?

(A) General Insurance Corpora-

tion Ltd.

(B) National Insurance Corn-

pany Ltd.

(C) Agriculture Insurance Com-

pany Ltd,

(D) SBI Life Insurance Company


Ltd.

(E) None of these

Answer.A

17. Who amongst the following is

new Foreign Secretary of India 7

(A) Nirupama Rao

(B) Goolarn E. Vahanvati

(C) Brindeshwar Pathak

(D) Naveen Chawla


(E) None of these

v

Answer.A

18. Which of the following correctly

describes what the ’Bancass-

urance’ is

1. It is an arrangement where-


by the branches of a bank

sell / distribute insurance

products of an insurance

company.

2. It is a new product devel-

oped/ launched by the banks

in which the risks of the

high value customers are

covered for any losses to


their property and/or lives

through an insurance cover.

3, It is a new product launched

by some banks by which

they are providing insurance

cover to exporters/ impor-

ters for the losses, if any due

to high fluctuations in the

exchange rates of dollars


and other major currencies.

(A) Only 1

(B) Only2

(C) Only 3

(D) All 1, 2 and 3

(E) None of these

Answer.A

19, Which of the following is the

abbreviated name of the organizations which provides credit


guarantee to exporters ?

(A) FICCI

(B) NABARD

(C) SIDBI

(D) ECGC

(E) SEBI

Answer.D

20. The Railway Budget 2009-10 was

presented in the Lok Sabha (in


Iuly 2009) by

(A) Lalu Prasad Yadav

(B) Kapil Sibal

(C) P. Chidambaram

(D) Mamata Banerjee

(E) Pranab Mukherjee

Answer.D

21. ‘Satyam’ the well known company which has been i.n the news


in recent past, is now known

with its new name as—

(A) Mahindra Satyarn

(B) Birla Satyam

(C) Reliance Satyam

(D) Tata Satyam

(E) None of these

Answer.A

22. The Economic Survey which was


presented in the Lok Sabha in

]uly 2009, was the survey for the

year/ period of—

(A) 2007-08

(B) 2008—09

(C) 2009-10

(D) Tenth five year plan

(E) None of these


Answer.B

23. The Government of India has

recently decided to allow about

2,200 items to come to Indian

markets with a tag of ‘Duty Free

Items} These items are imported

from

(A) China


(B) Singapore

(C) Sri Lanka

(D) Nepal

(E) Pakistan

Answer.B

24. Who amongst the following is

not one of the HFA Award

winners of 2009 ? (Under any one

of the categories).


(A) Priyanka Chopra

(B) Asin

(C) Farhan Akhtar

(D) Karishma Kapoor

(E) Hrithik Roshan

Answer.D

25. Saina Nehwal is a famous—

(A) Badminton player

(B) Table Tennis Player


(C) Lawn Tennis Player •

(D) Hockey Player

(E) Chess Player

Answer.A

26. ’No Tobacco Day’ is observed

every year on——

(A) lst May


(B) lst ]une

(C) 31st May

(D) 30th june

(E) None of these

Answer.C

27, India has recently appointed a

new Attorney General. Attorney

Generals are advisers to the

Government of India in-


(A) Defence related matters

(B) Diplomatic matters

(C) Legal matters

(D) Financial matters

(E) None of these

Answer.C

28. The presidential election in

which of the following countries,

held in ]une 2009, created a con•


troversy as one of the candidates

who was declared defeated

refused to accept the results of

the same ?

(A) Iran

(B) Pakistan

(C) Nepal

(D) Sri Lanka

(E) USA


Answer.A

29. Which of the following awards is

given for excellence in the field

of Literature 7

(A) N. Borlaug Award

(B) Kalinga Prize

(C) Kishor Kumar Award

(D) Saraswati Samman

(E) Dronacharya Award


Answer.D

30. The parliamentarian elections of

which of die following organizations were held in june 2009 ?

(A) European Union (EU)

(B) Oil & Petroleum Exporting

Countries (OPEC)

(C) Shanghai Corporation

(D) South Asian Association for


Regional Cooperation (SAARC)

(E) None of these

Answer.A

31. The 79th Amendment to the

Indian Constitution done in 1999

was associated with the»-

(A) reservation of seats in the

Parliament and State Assemblies


for sC/ ST candidates

(B) creation of three new states

Chhattisgarh, Uttarakhand and

jharkhand

(C) land reforms in many states

(D) giving more powers to

Panchayats

(E) None of these

Answer.A


32. Which of the following books is

written by Arundhati Roy ?

(A) The Last Hero

(B) Stolen Harvest

(C) A Himalayan Love Story

(D) Freedom From Fear

(E) God of Small Things

Answer.E

33. Which of the following schemes


was launched recently to make

Indian cities free from slums ?

(A) Bharat Nirrnan

(B) Indira Vikash Yojana

(C) Indira Awas Yojana

(D) Rajiv Awas Yojana

(E) None of these

Answer.D

34. Admiral Gorshkov Aircraft


carrier Refit Project is a deal

between India and-

(A) China

(B) South Korea

(C) Russia

(D) Iran

(E) None of these

Answer.C

35. The sex ratio in India is higher in


which of the following states ?

(Females (1052) are more than

per thousand males)-

(A) Maharashtra

(B) Kerala

(C) Manipur

(D) Assam

(E) Karnataka

Answer.B


36. Which of the following countries

in Not a member of ASEAN ?

(A) Myanmar

(B) Singapore

(C) China

(D) Vietnam

(E) Malaysia

Answer.C

37. Tipaimukh Dam which was in


news recently is located in which

of the following states ?

(A) Nagaland

(B) Meghalaya

(C) Arunachal Pradesh

(D) Manipur

(E) Assam

Answer.D

38. Which of the following states


has highest Literacy Rate in

country ?

(A) Gujarat

(B) Maharashtra

(C) Delhi

(D) Andhra Pradesh

(E) Kerala

Answer.E

39. Which of the following cups/


trophies is closely associated with

the game of Hockey ?

(A) Mardeka Cup

(B) Wimbledon Trophy

(C) Ranji Trophy

(D) Durand Cup

(E) Agha Khan Cup

Answer.E

40. Which of the following diseases


is caused by a virus ?

(A) Plague

(B) Cholera

(C) Whooping cough

(D) Tetanus

(E) Chicken pox

Answer.E
**********************************************
1. If the following numbers are rewritten by interchanging the digits in ten’s place and hundred’s place and then arranging them in the descending order. What will be the second digit of the newly formed fifth number from your right ?

479, 736, 895, 978, 389, 675

(A) 3

(B) 4

(C) 5

(D) 6

Ans : (C)




2. P is 60 m South-East of Q. R is 60 m North-East of Q. Then R is in which direction of P ?

(A) North

(B) North-East

(C) South

(D) South-East

Ans : (A)



Directions—(Q. 3–5) Read the following information for answering the questions that follow—


On a playing ground A, B, C, D and E are standing as described below facing the North.

(i) B is 50 metres to the right of D.

(ii) A is 60 metres to the South of B

(iii) C is 40 metres to the West of D.

(iv) E is 80 metres to the North of A.



3. If a boy walks from C, meets D followed by B, A and then E, how many metres has he walked if he has travelled the straight distance all through ?

(A) 120

(B) 150


(C) 170

(D) 230

Ans : (D)



4. What is the minimum distance (in metre approximately) between C and E ?

(A) 53

(B) 78

(C) 92

(D) 120


Ans : (C)



5. Who is to the South-East of the person who is to the left of D ?

(A) A

(B) B

(C) C

(D) E

Ans : (A)






6. A man was walking in the evening just before the sun set. His wife said that, his shadow fell on his right. If the wife was walking in the opposite direction of the man, then which direction the wife was facing ?

(A) North

(B) West

(C) South

(D) East

Ans : (C)



Directions—(Q. 7–11) In each of the following questions choose the set of numbers from the four alternative sets that is similar to the given set.




7. Given set : (4, 9, 18)

(A) (8, 14, 22)

(B) (10, 15, 25)

(C) (6, 12, 23)

(D) (12, 17, 26)

Ans : (D)



8. Given set : (10, 14, 17)

(A) (4, 11, 14)


(B) (9, 12, 15)

(C) (8, 13, 18)

(D) (6, 9, 12)

Ans : (A)



9. Given set : (7, 27, 55)

(A) (21, 35, 52)

(B) (18, 42, 65)

(C) (16, 40, 72)


(D) (13, 30, 58)

Ans : (C)



10. Given set : (39, 28, 19)

(A) (84, 67, 52)

(B) (52, 25, 17)

(C) (70, 49, 36)

(D) (65, 45, 21)

Ans : (A)




11. Given set : (246, 257, 358)

(A) (233, 343, 345)

(B) (273, 365, 367)

(C) (143, 226, 237)

(D) (145, 235, 325)

Ans : (A)



Directions—(Q. 12–16) Each question contains six or seven statements followed by four sets of combinations of three. Choose the set in which the statements are logically related.




12. (1) All books are having pages.

(2) All kings are having pages.

(3) All kings are books.

(4) Some heavy things are having pages.

(5) Some heavy things are books.

(6) Some books are heavy.

(7) Some heavy things are having pages.

(A) 1, 2, 3


(B) 6, 1, 4

(C) 4, 6, 1

(D) 1, 5, 7

Ans : (D)



13. (1) No athletes are vegetarians.

(2) All cricket players are athletes.

(3) Some athletes play cricket.

(4) Some cricket players are vegetarians.


(5) No cricket player is a vegetarian.

(6) All athletes are vegetarians.

(A) 1, 2, 5

(B) 3, 4, 1

(C) 1, 5, 2

(D) 2, 5, 6

Ans : (A)



14. (1) All grandmothers cook well.


(2) No man is a grandmother.

(3) Some men do not cook well.

(4) All those who cook well are men.

(5) No one who cooks well is a man.

(6) All those who cook well are grandmothers.

(7) Some men are not grandmothers.

(A) 2, 6, 5

(B) 2, 5, 6

(C) 1, 4, 2


(D) 6, 4, 7

Ans : (B)



15. (1) Looting is a crime.

(2) Some crooked people are criminals.

(3) All those involved in looting are criminals.

(4) Some crooked people are involved in looting.

(5) All criminals are looked down in society.

(6) Some crooked people are not criminals.


(A) 1, 4, 6

(B) 3, 6, 2

(C) 1, 2, 6

(D) 3, 4, 2

Ans : (D)



16. (1) Some women are those who are successful in life.

(2) Some men are those who have patience.

(3) No man is a woman.


(4) All those who have patience are successful in life.

(5) Some who are successful in life are men.

(6) Some men are not those are successful in life.

(A) 1, 3, 6

(B) 4, 2, 6

(C) 1, 5, 3

(D) 2, 4, 5

Ans : (B)




Directions—(Q. 17–21) Each of the questions below consists of a question and two statements numbered (I) and (II). You have to decide whether the data provided in the statements are sufficient to answer the question. Give answers—

(A) If the data in statement (I) alone are sufficient to answer the question, while the data in statement (II) alone are not sufficient to answer the question;

(B) If the data in statement (II) alone are sufficient to answer the question, while the data in statement (I) alone are not sufficient to answer the questions;

(C) If the data even in both statements (I) and (II) together are not sufficient to answer the question;

(D) If the data in both statement (I) and (II) together are necessary to answer the question.



17. In which direction is Mahatmaji’s statue facing ?

I. The statue is towards the northern end of the city.


II. The statue’s shadow falls towards East at 5 O’clock in the evening.

Ans : (C)



18. What is the total number of pupils in the final year class ?

I. The number of boys in the final year class is twice as much as the number of girls in that class.

II. The sum of the ages of all the pupils in the class is 399 years and their average age is 19 years.

Ans : (B)



19. Who is the tallest among A, B, C and D ?


I. A is taller than C.

II. B is taller than C and D.

Ans : (C)



20. How many Sundays are there in a particular month of a particular year ?

I. The month begins on Monday.

II. The month ends on Wednesday.

Ans : (D)



21. What is the total number of pages in this book ?


I. I counted 132 pages from the beginning of this book.

II. My wife counted 138 pages starting from the end of the same book.

Ans : (C)



Directions—(Q. 22–26) In each of the questions given below, there is a statement followed by three assumptions numbered I, II and III. An assumption is something supposed or taken for granted. You have to consider the statement and assumptions and then decide, which of the assumption(s) is/are implicit in the statement.



22. Statement : During pre-harvest kharif seasons, the government has decided to release vast quantity of foodgrains from FCI.


Assumptions :I. There may be a shortage of foodgrains in the market during this season.

II. The kharif crop may be able to replenish the stock of FCI.

III. There may be a demand from the farmers to procure kharif crop immediately after harvest.

(A) None is implicit

(B) Only I and II are implicit

(C) Only II and III are implicit

(D) All are implicit

Ans : (D)




23. Statement : To improve the employment situation in India, there is a need to recast the present educational system towards implementation of scientific discoveries in daily life.

Assumptions :I. The students after completing such education may be able to earn their livelihood.

II. This may bring meaning of education in the minds of the youth.

III. The state may earn more revenue as more and more people will engage themselves in self employment.

(A) Only I and II are implicit

(B) Only III is implicit


(C) Only I and III are implicit

(D) None is implicit

Ans : (A)



24. Statement : To increase profit, the oil exporting countries decided to reduce the production of crude by 5 million barrels per day.

Assumptions :I. The price of crude may increase due to less production.

II. The demand of crude may remain same in future.


III. Other countries may continue buying crude from these countries.

(A) All are implicit

(B) Only II and III are implicit

(C) Only I and II are implicit

(D) None is implicit

Ans : (C)



25. Statement : “We do not want you to see our product on newspaper, visit our shop to get a full view.” – an advertisement.


Assumptions :I. People generally decide to purchase any product after seeing the name in the advertisement.

II. Uncommon appeal may attract the customers.

III. People may come to see the product.

(A) All are implicit

(B) None is implicit

(C) Only II and III are implicit

(D) Only I and II are implicit

Ans : (A)




26. Statement : The Reserve Bank of India has directed the banks to refuse fresh loans to major defaulters.

Assumptions :I. The banks may still give loans to the defaulters.

II. The defaulters may repay the earlier loan to get fresh loan.

III. The banks may recover the bad loans through such harsh measures.

(A) All are implicit

(B) None is implicit


(C) Both II and III are implicit

(D) Both I and II are implicit

Ans : (C)



Directions—(Q. 27–31) In questions given below, statements 1 and 2 are followed by conclusions I and II. Taking the statements to be right although they may seem at variance with commonly accepted facts, mark your answers as under—

(A) If only conclusion I follows.

(B) If only conclusion II follows.

(C) If both I and II follows.


(D) Neither I nor II follows.



27. Statements :1. All hands are machines.

2. All machines are wheels.

Conclusions :I. All wheels are hands.

II. All hands are wheels.

Ans : (B)




28. Statements :1. Some buds are leaves.

2. Some leaves are red.

Conclusions :

I. Some buds are red.

II. Some leaves are not buds.

Ans : (B)



29. Statements :


1. Some stones are shells.

2. All shells are pearls.

Conclusions :I. Some stones are pearls.

II. All pearls are shells.

Ans : (A)



30. Statements :1. Brown is red and blue is green.

2. Green is pink and yellow is red.


Conclusions :

I. Yellow is brown.

II. Pink is blue.

Ans : (C)



31. Statements :

1. Merchants who do not own cars do not have bicycles either.

2. Those who do not have bicycles have tricycles.


Conclusions :I. Some merchants have only tricycles.

II. No one has both, the car and the tricycles.

Ans : (D)



Directions—(Q. 32–36) A number arrangement machine, when given a particular input, rearranges it following a particular rule. The following is the illustration of the input and the stages of arrangement.

Input : 245, 316, 436, 519, 868, 710, 689

Step I : 710, 316, 436, 519, 868, 245, 689


Step II : 710, 316, 245, 519, 868, 436, 689

Step III : 710, 316, 245, 436, 868, 519, 689

Step IV : 710, 316, 245, 436, 519, 868, 689

Step IV is the last step of input.



32. If 655, 436, 764, 799, 977, 572, 333 is the input which of the following steps will be ‘333, 436, 572, 655, 977, 764, 799’ ?


(A) II

(B) III

(C) IV

(D) I

Ans : (B)



33. How many steps will be required to get the final output from the following input ?

Input : 544, 653, 325, 688, 461, 231, 857


(A) 5

(B) 4

(C) 3

(D) 6

Ans : (A)



34. For the given input, which of the following will be third step ?

Input : 236, 522, 824, 765, 622, 463, 358


(A) 522, 236, 765, 824, 622, 463, 358

(B) 522, 622, 236, 824, 765, 463, 358

(C) 522, 622, 236, 765, 824, 463, 358

(D) 522, 622, 236, 463, 824, 765, 358

Ans : (C)



35. If following is the second step for an input, what will be the fourth step ?

Step II : 620, 415, 344, 537, 787, 634, 977


(A) 620, 415, 344, 537, 634, 787, 977

(B) 620, 415, 344, 634, 537, 787, 977

(C) 620, 415, 344, 634, 787, 537, 977

(D) Can’t be determined

Ans : (B)



36. Which of the following is the last step for the following input ?

Input : 473, 442, 735, 542, 367, 234, 549


(A) 234, 442, 542, 473, 735, 367, 549

(B) 234, 442, 542, 735, 473, 367, 549

(C) 234, 442, 542, 473, 367, 735, 549

(D) 234, 442, 542, 735, 367, 473, 549

Ans : (A)



Directions—(Q. 37–41) Read the following information carefully and answer the questions given below it—

(1) There is a group of six persons A, B, C, D, E and F in a family. They are Psychologist, Manager, Lawyer, Jeweller, Doctor and Engineer.


(2) The doctor is the grandfather of F, who is a Psychologist.

(3) The Manager D is married to A.

(4) C, the Jeweller, is married to the Lawyer.

(5) B is the mother of F and E.

(6) There are two married couples in the family.

(7) The Psychologist is a female while Engineer is a male.



37. What is the profession of E ?

(A) Doctor


(B) Engineer

(C) Manager

(D) Psychologist

Ans : (B)



38. How is A related to E ?

(A) Brother

(B) Uncle

(C) Father


(D) Grandfather

Ans : (D)



39. How many male numbers are there in the family ?

(A) One

(B) Three

(C) Four

(D) Two

Ans : (B)




40. What is the profession of A ?

(A) Doctor

(B) Lawyer

(C) Jeweller

(D) Manager

Ans : (A)



41. Which of the following is one of the pairs of couples in the family ?

(A) AB


(B) AC

(C) AD

(D) AE

Ans : (C)



42. Three of the following four are alike in a certain way and so form a group. Which is the one that does not belong to that group ?

(A) Papaya

(B) Mango

(C) Jackfruit


(D) Watermelon

Ans : (C)



43. Three of the following four are similar in relation to their positions in the English alphabet and hence form a group. Which one does not belong to that group ?

(A) SPEAK : PZVKH

(B) HUSKY : BPGFS

(C) BRAIN : MRZIY

(D) BREAK : PZVIY

Ans : (B)




44. Three of the following four groups of letters are alike in some way while one is different. Find out which one is different ?

(A) DJWR

(B) EKXR

(C) KQDX

(D) AGTN

Ans : (A)



45. Four groups of letters are given below. Three of them are alike in a certain way while one is different. Choose the odd one.

(A) GWOURV


(B) LZKMSU

(C) SFXPMG

(D) JOEHNP

Ans : (C)



46. Four pairs of words are given below out of which the words in all pairs except one, bear a certain common relationship. Choose the pair in which the words are differently related.

(A) Watt : Power

(B) Ampere : Current


(C) Pascal : Pressure

(D) Radian : Degree

Ans : (D)



47. Number of letters skipped in between adjacent letters of the series starting from behind are increased by one. Which of the following series observes this rule ?

(A) ONLKI

(B) OMKIG

(C) OMJFA

(D) OIGDC


Ans : (C)



48. If the letters of the word ‘PROTECTION’ which are at odd numbered position in the English alphabet are picked up and are arranged in alphabetical order from left and if they are now substituted by Z, Y, X and so on, beginning from left which letter will get substituted by X ?

(A) E

(B) O

(C) T

(D) I

Ans : (D)




49. How many pairs of letters are there in the word OPERATION in which the difference between them is the same as in the English alphabet ?

(A) 3

(B) 5

(C) 7

(D) 9

Ans : (C)



50. Arrange the given words in order in which they occur in the dictionary and then choose the correct sequence.

(1) Dissipate


(2) Dissuade

(3) Disseminate

(4) Distract

(5) Dissociate

(6) Dissect

(A) 6, 3, 1, 5, 2, 4

(B) 1, 6, 3, 2, 4, 5

(C) 3, 6, 1, 2, 5, 4

(D) 4, 6, 3, 1, 5, 2


Ans : (A)



51. The letters skipped in between the adjacent letters in the series are followed by equal space. Which of the following series observes this rule ?

(A) SUXADF

(B) RVZDFG

(C) HKNGSW

(D) RVZDHL

Ans : (D)



Directions—(Q. 52–56) In a certain code, letters of English alphabet are coded as given for some words. The numeric code for each letter is given in bracket under coded form and corresponds to the letter in the word in the same serial order, study the coded forms of the given words and find out the rules for their classification. Applying those rules find out the code for the words given in capital letters in the questions that follow—


Word Codes FormATE (0) (5) (0)

NONE (5) (25) (5) (25)

UNIT (30) (5) (30) (5)

PIN (5) (10) (5)

PAGE (5) (25) (5) (25)

OPEN (30) (5) (30) (5)

ONE (0) (5) (0)


CUT (5) (10) (5)

SEAT (5) (15) (15) (5)

DEEP (5) (20) (20) (5)



52. VINA

(A) (5) (0) (5) (15)

(B) (5) (25) (5) (25)

(C) (5) (30) (5) (30)

(D) (5) (10) (5) (30)


Ans : (B)



53. AGE

(A) (0) (15) (0)

(B) (15) (15) (15)

(C) (0) (10) (10)

(D) (0) (5) (0)

Ans : (D)



54. PEAR


(A) (5) (15) (15) (5)

(B) (5) (25) (5) (25)

(C) (5) (10) (5) (10)

(D) (5) (30) (5) (30)

Ans : (A)



55. TIN

(A) (0) (5) (0)

(B) (5) (0) (5)


(C) (0) (10) (0)

(D) (5) (10) (5)

Ans : (D)



56. UNIT

(A) (5) (30) (5) (30)

(B) (5) (10) (30) (10)

(C) (30) (5) (30) (5)

(D) (15) (10) (10) (15)


Ans : (C)



57. If the first and second digits in the squence 7 9 8 4 5 3 6 7 8 3 4 5 are interchanged, also the third and fourth digits, the fifth and sixth digits and so on which digit would be the sixth counting from your right ?

(A) 5

(B) 6

(C) 7

(D) 8

Ans : (C)



58. The letter I, J, K, L, M, N, O, P, Q, R, S, T in their order are substituted by twelve numbers but not in that oder. 3 is assigned to R. The difference between R and M is 7. The difference between K and M is 2. What number is assigned to K ?


(A) 8

(B) 12

(C) 7

(D) 11

Ans : (B)



59. Below are given six three-character numbers. The characters comprise of digits and letters. The letter stands for one less than its serial order in the English alphabet. What will be the middle character of the 3rd number when the numbers are arranged in the decending order ?

8G6, 3DJ, F4C, 7HB, 4E6, B8I

(A) 7


(B) 8

(C) 6

(D) 4

Ans : (D)



60. Which of the following will be the third digit of the fourth number after the following numbers are arranged in descending order after reversing the positions of the digits within each number ?

645, 869, 458, 347, 981, 792

(A) 4

(B) 6


(C) 8

(D) 9

Ans : (A)
*********************************************
1. Which of the following stars lies nearest to our solar system ?

(A) Barnard’s Star

(B) Sirius A

(C) Alpha Centauri A

(D) Proxima Centauri

Ans : (D)



2. Of the total water on the earth, fresh water reserves constitute approximately—

(A) 5•8%


(B) 4•5%

(C) 2•59%

(D) 1•2%

Ans : (C)



3. The term ‘Bishop’ in sports is related to—


(A) Chess

(B) Hockey

(C) Badminton

(D) Bridge

Ans : (A)



4. Yakshagan is the famous dance form of the State of—

(A) Orissa

(B) Andhra Pradesh


(C) Tamil Nadu

(D) Karnataka

Ans : (D)



5. Balance of Payments means—

(A) Difference between export and imports

(B) Balance to be paid to the exporters

(C) Balance to be paid to the industrialists

(D) Balance coming in allocation of funds for States


Ans : (A)





6. A light year is equivalent to about—

(A) 365 days

(B) Six million miles

(C) Six billion miles

(D) Six trillion miles

Ans : (D)




7. Denmark, Iceland, Norway, Sweden and Finland are together called—

(A) Slovakia

(B) Scandinavia

(C) Netherlands

(D) Australasia

Ans : (B)



8. The device used for locating submerged objects under sea is—

(A) Sonar


(B) Radar

(C) Laser

(D) Maser

Ans : (A)



9. Which article of the Constitution provides the Parliament the power to ammend Constitution ?

(A) 376

(B) 370

(C) 368


(D) 390

Ans : (C)



10. What is the name of the first successfully cloned deer ?

(A) Dawn

(B) Deluxe

(C) Demor

(D) Dewey

Ans : (D)




11. The idea of motion pictures was propounded by—

(A) N. R. Finsen

(B) T. A. Edison

(C) J. L. Baird

(D) Berliner

Ans : (B)



12. In Greek mythology, Apollo is the god of which of the following ?

(A) Love


(B) Peace

(C) Prophecy

(D) Medicine

Ans : (C)



13. NASA’s new space telescope is—

(A) Wise

(B) Rise

(C) Barack


(D) Telle

Ans : (A)



14. Bull fighting is the national game of—

(A) Italy

(B) Poland

(C) Spain

(D) Sudan

Ans : (C)




15. In a rare recognition, the Government of Mauritius decides to introduce a book written by Uttarakhand Chief Minister Ramesh Pokhriyal Nishank in the School Syllabus of the island nation, what is the name of the book ?

(A) Sound of Hills

(B) Sparsh Ganga

(C) Heaven

(D) Amrit

Ans : (B)



16. Who has been selected for the ‘Niwano Peace Prize, 2010’ for her contribution to the uplift of poor women in India ?


(A) Ela Bhatt

(B) Jhumpa Lahiri

(C) Punam Suri

(D) Mandakni Apte

Ans : (A)



17. On which of the following dates in 2010 did the safeguard agreement with regard to civilian nuclear facilities between India and the International Atomic Energy Agency (IAEA) come into force ?

(A) February 28

(B) February 21


(C) February 11

(D) February 4

Ans : (D)



18. The tax which the kings used to collect from the people in the Vedic period was called—

(A) Bali

(B) Vidatha

(C) Varman

(D) Kara


Ans : (B)



19. The cells which are responsible for the production of antibodies are—

(A) Red blood cells

(B) Neutrophils

(C) Lymphocytes

(D) Platelets

Ans : (C)



20. Correct expansion of the term ‘http’ in Internet address is—


(A) Hybrid text transfer protocol

(B) Hyper text transfer protocol

(C) Higher transfer text protocol

(D) Higher text transfer protocol

Ans : (B)
*******************************************************


General Awareness (Held on 10-08-2008)



1. The foreign trade policy announced in the year 2004 was announced for a period of—

(A) two years

(B) three years

(C) four years

(D) five years

(E) ten years

Ans : (D)



2. As per reports in the newspapers the Indian Rupee is appreciating these days. What does it really mean it ?


(1) The value of the Rupee has gone up. It is now 110 paise and not 100 paise.

(2) The exchange rate of Rupee has gone up.

(3) Now we can purchase more in one Rupee which was not possible earlier.

(A) Only 1

(B) Only 2

(C) Only 3

(D) Both 1 & 2

(E) None of these


Ans : (B)



3. As per newspaper reports the inflation in India and China was at very high level . In such a situation the central Banks of these countries are required to follow—

(A) a more liberal credit policy

(B) a very tight credit policy

(C) create an atmosphere of easy liquidity in the market.

(D) raise the limits of personal and corporate income taxes.

(E) None of these

Ans : (B)




4. The US Federal Reserve had recently reduced the interest rates by 150 basis points in the last few months. This was done because—

(A) US was going into recession.

(B) Inflation was high in countries like India and China hence a correction in economic policies by all the countries was needed.

(C) Euro had become more attractive currency, hence US wanted to save dollars from further depreciation.

(D) Iraq had bacome a very lucrative destination for foreign investment . US does not want it to happen.

(E) None of these

Ans : (A)



5. The Stock market index of London Stock Market is referred as—


(A) Sensex

(B) Footsie (FTSE)

(C) NIFTY

(D) Bullish

(E) None of these

Ans : (B)



6. As we all know Ministry of Finance every year prepare Union Budget and present it to the parliament . Which of the following is / are the elements of the Union Budget ?

(1) Estimates of revenue and capital receipts.


(2) Ways and MeAns : to raise the revenue.

(3) Estimates of expenditure.

(A) Only 1

(B) Only 2

(C) Only 3

(D) All 1, 2 & 3

(E) None of these

Ans : (D)




7. Which of the following is NOT used as raw material for production of nuclear power?

(A) Uranium

(B) Barylium

(C) Zirconium

(D) Sodium

(E) All these are used

Ans : (A)



8. As per figure released recently, the GDP growth in the last two years has been in the range of—


(A) 6%–7%

(B) 7%–8%

(C) 8%–9%

(D) 9%–9.5%

(E) 10%–11%

Ans : (D)




9. Sri Lankan Monitoring Mission (SLMM) has Nordic Monitors from five nations. Which of the following is NOT one of them?

(A) India

(B) Sweden

(C) Norway

(D) Denmark

(E) Iceland

Ans : (A)



10.Many a times we read a term in financial newspaper GDR. What is the full form of the same?


(A) Gross Domestic Revenue

(B) Global Domestic Ratio

(C) Global Depository Receipts

(D) Gross Depository Revenue

(E) None of these

Ans : (C)



11. The Govt. of India has fixed some targets for various programmes for the entire plan period/or a part of it. Which of the following is/are TRUE about the Bharat Nirman Programme?

(1) The plan envisages irrigation of 10 million hectares additional land.


(2) It is planned that about six hundred km. of new roads will be developed every month.

(3) About 1000 new telephone connections will be given to rural areas every month so that by the end of the plan all villages in India are brought under telephone services network.

(A) Only (1) is true

(B) Only (2) is true

(C) Only (3) is true

(D) All (1), (2) & (3) are true

(E) None is true

Ans : (A)


12. Who played the role of the Hockey coach in the Film 'Chak De India' for which he was given the award for Best Actor?

(A) Shahrukh Khan

(B) Abhishek Bachchan

(C) Aaamir Khan

(D) Salman Khan

(E) None of these

Ans : (A)



13. India has signed a Memorandum of Understanding (MOU) with which of the following countries which says they will create a "Multi Polar Order" based on the rule of law, sovereign equality and non-interference in internal affairs of States?


(A) USA

(B) Nepal

(C) China

(D) Britain

(E) Russia

Ans : (E)



14. Recently one Indian cricket player has become the sixth batsman to earn 10000 runs in Test Crickets. The name of the player is—

(A) Virendra Sehwag


(B) Sachin Tendulkar

(C) Sourav Ganguly

(D) Anil Kumble

(E) Rahul Dravid

Ans : (E)



15. Yousuf Raza Gillani who was elected the Prime Minister of Pakistan is from which political parties?

(A) Jamiat-e-Ulema Islami

(B) Pakistan People's Party


(C) Awami National Party

(D) Muslim League

(E) None of these

Ans : (B)



16. As per the Govt. Notifications / Advertisements given in various newspapers the wheat procurement this year was of a record of 210 lakh MTs. Which of the following was/were the contributory factors in such a record making procurement?

(1) Minimum support price was keep as Rs. 1,500/- per quintal.

(2) Wheat was also procured from non-traditional State like Gujrat and Maharashtra

(3) State Govt. were also involved actively in the Scheme


(A) Only 1

(B) Only 2

(C) Only 3

(D) Both 1 & 2

(E) None of these

Ans : (D)



17. Very often we read in newspapers that some sports persons have to go for a Dope Test after their performance in the field. Why Dope Test is conducted ?


(1) To know the general health of the sports person

(2) To know the sugar level of their blood

(3) To detect whether they have taken any performance inhancing drugs

(A) Only 1

(B) Only 2

(C) Only 3

(D) All 1, 2 & 3

(E) None of these


Ans : (B)



18. Santosh Trophy is associated with the game of—

(A) Cricket

(B) Badminton

(C) Tennis

(D) Chess

(E) None of these

Ans : (E)
Hint : Santosh Trophy is associated with the game of Football.




19. As per the newspaper reports some economically developed states onlt hold about 60% of the total 'Demat Accounts in India. A Demat Account is—

(A) an acount which is opened by the people of the lower income groups of the society.

(B) an account in which trading of the shares is done.

(C) an account which can be opened only by minors.

(D) an account which can be operated by big corporate houses and are mainly business accounts like current accounts.

(E) None of these

Ans : (B)

20. Federal Reserve is the Financial Organization of the—


(A) USA

(B) Britain

(C) France

(D) Japan

(E) Germany

Ans : (A)

21. Which of the following organization is known as Market Regulator in India?

(A) IBA

(B) SEBI


(C) AMFI

(D) NSDL

(E) None of these

Ans : (B)



22. Which of the following phenomenon occurred /surfaced recently associated with the economy of USA has put a catastrophic influence on the economy of the World?

(A) Increase in the oil prices

(B) High level of inflation

(C) Huge and mounting expenditure due to attack on Iraq by USA and allied forces.


(D) Sub-prime Crisis

(E) None of these

Ans : (D)



23. The Reserve Bank of India keeps on changing various ratio/rates frequently. Why this is done?

(1) To keep inflation under control.

(2) To ensure that Indian Rupee does not loose its market value.

(3) To ensure that Banks do not earn huge profits at the cost of public money.

(A) Only 1


(B) Only 2

(C) Only 3

(D) All 1, 2 & 3

(E) None of these

Ans : (D)



24. Which of the following is true about the National Rural Employment Guarantee Act?

(1) It is applicable only in 100 rural districts.


(2) It gives a guaranteed employment of 100 days to all those who are eligible for the same and are also willing to take it.

(3) This Act is applicable only for men between the age of 21-65. Women do not get the benefit of the same.

(A) Only 1

(B) Only 2

(C) Only 3

(D) All 1 , 2 & 3

(E) None of these

Ans : (B)




25. Many times we here about SEZ in newspapers/magazines. What is the full form of SEZ?

(A) Small Economic Zone

(B) Social Economic Zone

(C) Special Enforcewment Zone

(D) Service & Economic Zone

(E) None of these

Ans : (E)

****************************************************

Directions: In this Section you have Six short passages. After each passage you will find several questions based on the passage. First, read Passage-I, and answer the questions based on it. Then go on to the other passages.



PASSAGE—I

The martyrs who laid down their lives for the freedom of the country, had a lofty vision of the future. They wanted the nation to be free from all the slavery and bondage. They wanted an India in which all the communities would live in perfect harmony and in which there would be no high class and no low class of people, the curse of untouchability having been wiped out completely. Women would enjoy equal rights with men and contribute their fullest to the making of a great nation. Such a vision was in keeping with the ancient glory of the country renowned for its splendid achievements in literature, art and culture.

We must now revitalise this ancient culture of ours with tolerance as its masthead. If we forget or cease to take pride in our noble heritage, we shall have to face severe indictment in the court of history which is a ruthless judge and seldom spares the erring people.


1. The martyrs who died for the freedom of India wanted:

(a) the country to be the strongest nation in the world

(b) the country to rule over the other nations

(c) the country to be free from slavery

(d) the people to give up their antiquated customs



2. These martyrs wanted that:

(a) there should be reservation in the jobs for the backward sections of the society

(b) there should be perfect communal love and peace in the country


(c) the old caste-system should be retained in the future

(d) the women should look after their families only



3. We must strive with our total commitment to:

(a) defeat and overcome the enemies of the nation

(b) revitalise our rich past culture

(c) inject scientific temper into our past culture

(d) make scientific advancements



4. Our freedom-fighters envisioned that in free India:


(a) there would be an egalitarian society

(b) women would enjoy higher privileges and rights than others

(c) the country would be taken forward by some selected classes of the society

(d) industrialisation should occupy the top priority



PASSAGE—II

Dr S. Radhakrishnan, the illustrious philosopher statesman of India, was one of the greatest sons of our motherland. He cautioned the world against the domination of science in society. It is erroneous to claim that scientific knowledge would bring with it perpetual progress and a steady improvement in human relations. The recent period of great scientific achievements has also increased human misery: two world wars, concentration camps, atomic destruction, cold war, deadly wars in the middle east, Persian Gulf and at many other places in the world. Growth in human wisdom has not been commensurate with the increase in scientific knowledge and power. The fear of universal destruction hangs over the world. There is a feeling of disenchantment, anxiety and even despair. Science has failed to liberate man from the tyranny of his own nature.

Mankind is passing through a critical period and an education of the human spirit has become essential. In order to remake society, man has to remake himself. If humanity is to survive, man must integrate his knowledge with a social responsibility.




5. Dr S. Radhakrishnan has:

(a) emphasised that science should be banished from the society

(b) opposed the teaching of science in educational institutions

(c) favoured scientific thinking in life

(d) counselled that preponderance of science in life does not necessarily generate happiness



6. The recent past of tremendous scientific progress has:

(a) made the world a very happy place

(b) led to global warming


(c) brought about internal transformation in men

(d) shown that human wisdom has not kept pace with galloping scientific knowledge



7. Man is despaired of science because:

(a) science has given too much knowledge

(b) science has brought him excessive material comforts

(c) he has become a captive of science

(d) he is confronted with the nightmare of total annihilation of the world



. Man can save humanity only if he:


(a) abandons science

(b) brings about an internal transformation in himself

(c) makes his life more comfortable with scientific gadgets

(d) goes back to nature and primitive times



9. In this passage, the writer has tried to show that:

(a) science is the only saviour that shall lead humanity forward

(b) science can bring about an end to all the wars

(c) social change comes with the advancement of science


(d) human wisdom must grow proportionately with growth of knowledge to evolve a creative integration to help mankind





PASSAGE—III

In Asia and much of the Third World, trees are still destroyed in the old-fashioned way : they are cut down for fuel and cropland. In Europe, there is new and potentially more deadly culprit. The Germans call it ‘Waldsterben’, the dying forest syndrome. But the disease is far more than a German phenomenon. Since it was first observed by German scientists in the autumn of 1980, the mysterious malady has raced across Europe, blighting woods in countries as far apart as Sweden and Italy.

Explanations for the epidemic range from a cyclic change in the environment to a baffling form of tree cancer. But the most convincing evidence points to air pollution. Indeed, saving the rapidly deteriorating forests of Europe will probably require a two-pronged strategy : an offensive campaign that includes the breeding of pollution-immune trees and a defensive scheme that calls for reductions in toxic emissions. But both will require more money than is currently being spent on such measures, as well as total commitment to protecting the environment.



10. According to the passage, which one of the following statements is correct?

(a) There is less damage in Asia than in Europe


(b) More forests are dying in Germany than anywhere else in Europe

(c) A cyclic change in the environment is responsible for deforestation

(d) Air pollution is the main culprit of destroying European forests



11. Saving the tress of European forests:

(a) should not be difficult because of the advances in experimental research

(b) appears to be a hopeless task and therefore pointless to undertake

(c) requires a much bigger budget

(d) demands vigilance and punitive measures against those who cut down the trees




12. The dying forest syndrome is a disease that:

(a) is peculiar to the forests of Asia

(b) has spread rapidly over the forests of Europe

(c) is confined to the forests of Germany

(d) has affected forests all over the world



13. The writer suggests that:

(a) it is no longer possible to grow trees in industrialized areas

(b) pollution-immune trees wil absorb toxic emissions


(c) all pollution-prone trees should be destroyed

(d) it is not possible to grow trees that remain unaffected by pollution



14. The writer’s approach toward the problem of forest devastation is one of:

(a) tolerance

(b) indifference

(c) well thought-out strategy

(d) despondency




PASSAGE—IV

One of the major crises facing the country is the looming water shortage. A recent report of the UN has named India among the worst countries for poor quality of ater. The report ranks 122 countries according to  the quality of their water as well as their ability and commitment to improve the situation. Belgium is considered the worst basically because of the quality of its ground water.

Rains failed in most parts of India last year and the vast areas of Rajasthan, Madhya Pradesh, Andhra and Orissa were in the grip of devastating drought. People without water turn desperate and violent. Villagers in Rajasthan last year attacked the Food Corporation godowns.

Worse may be coming. With man refusing to control pollution (America, the world’s greatest polluter, refuses to cooperate with other countries) the world is getting hotter. This means that the great ice shelves (weighing billions of tonnes) of the Antarctic are collapsing. We cannot even conceptualise the dangerous consequences. Last century, sea levels in Venice rose by one step of a staircase. This century they are expected to rise by five steps. An additional cause for Venice’s sinking is the draining of underground water table due to industrialization.

The water tables in our cities have also been going lower and lower. When ocean level rises, Tuvalu in the Pacific Ocean will be the first to go under the waves. Citizens of that country are already migrating to New Zealand. Will citizens of Maldives crowd into Kerala? Will another mass migration from Bangladesh turn West Bengal upside down?



15. Citizens of Tuvalu are migrating to:


(a) Belgium (b) West Indies

(c) Morocco (d) New Zealand



16. Belgium, is suffering acutely because of:

(a) the sluggish pace of its economy

(b) the discharge of industrial effluents

(c) quality of its ground water

(d) rising cost of living



17. Villagers in Rajasthan attacked Food Corporation godowns because of:


(a) low prices offered to them for wheat

(b) refusal of Food Corporation to buy wheat from the local farmers

(c) no financial help from the government bodies

(d) shortage of water



18. One of the reasons for Venice’s sinking is:

(a) industrialization

(b) its proneness to earthquake

(c) felling of trees


(d) civil construction



PASSAGE—V

Most of us use the products of science—railways, aeroplanes, electricity, wireless and thousands of others—without thinking how did they come into existence. We take them for granted, as if we were entitled to them as a matter of right. And we are very proud of the fact that we live in an advanced age and are ourselves so very advanced. Now, there is no doubt that our age is a very different one from previous ages and I think it is perfectly correct to say that it is far more advanced. But that is a different thing from saying that e as individuals or groups are more advanced. It  would be the height of absurdity to say that because an engine driver can run an engine and Plato or Socrates could not, the engine driver is more advanced than, or is superior to, Plato or Socrates. But it would be perfectly correct to say that the engine itself is a more advanced method of locomotion than Plato’s chariot was.

19. Which one of the following statements is correct:

(a) An engine driver cannot be compared to Plato or Socrates


(b) Plato or Socrates is in no way inferior to the engine driver

(c) Plato or Socrates surpassed the engine driver in every respect

(d) An engine driver is cleverer than Plato or Socrates



20. People today are very proud because they:

(a) live in a philosophically advanced age

(b) live in a spiritually advanced age

(c) enjoy digital communications

(d) live in a scientifically advanced age




21. Many of us make use of machines:

(a) with full knowledge of their genesis

(b) without knowing how were they invented

(c) with very little knowledge of their mechanism

(d) without any knowledge of their historical significance



22. In this passage, the writer mentions Plato and Socrates to emphasise that:

(a) they had a great respect for learning

(b) they were men of great scholarship


(c) people as individuals in the modern age are not more advanced than their predecessors

(d) the engine is a better mode of locomotion than Plato’s chariot





PASSAGE—VI

To sum up the whole, we should say that the aim of the Platonic philosophy was to exalt man into a God. The aim of the Baconian philosophy was to provide man with what he requires while he continues to be a man. The aim of Platonic philosophy was to raise us far above the vulgar wants. The aim of Baconian philosophy was to supply our vulgar wants. The former aim was noble; but the latter was attainable. Plato drew a good bow; he aimed at the stars; and therefore, though there was no want of strength or skill, the shot was thrown away. His arrow was indeed followed by a track of dazzling radiance; but it struck nothing. Bacon fixed his eye on a mark which was placed on earth and hit it in the white. The philosophy of Plato began with words and ended in words, noble words indeed, words such as were to be expected from the finest of human intellects exercising boundless dominion over the finest of languages.



23. The above passage presents Platonic philosophy as:

(a) giving rise to volgar wants


(b) too idealistic in terms of a realistic assessment of a man

(c) no more than mere words

(d) being pragmatic



24. The author in the above passage:

(a) praises Platonic philosophy

(b) explains the drawbacks of Baconian philosophy

(c) balances opposite views in order to arrive at a just definition of each

(d) compares and contrasts two systems towards which he is impartial




25. Which one of the following best reflects the underlying tone of the passage?

(a) All ideas regarding man are couched in noble words

(b) Man when exalted into a God comes to nothing

(c) It is better for man to continue to be a man

(d) It is the image of man conceived differently that makes the basic distinction between different systems



26. Which of the following words in the passage confirm the exalted notions of man according to Plato?

(a) ‘exercising boundless dominion’


(b) ‘he aimed at the stars’

(c) ‘fixed his eye on the mark and hit it in the white’

(d) ‘there was no want of strength or skill, the shot was thrown away’

27. The reference to a bow and an arrow in the passage:

(a) serves as an example of Plato’s strength

(b) teaches us to stay focused on target

(c) shows us a track of dazzling brilliance in man’s history


(d) shows us the comparative strengths and weaknesses of Platonic and Baconian systems





SENTENCE IMPROVEMENT

Directions: Look at the underlined part of each sentence. Below each sentence are given three possible substitutions for the underlined part.



If one of them (a), (b) or (c) is better than the underlined part, indicate your response on the Answer Sheet against the corresponding letter (a), (b) or (c).



If none of the substitutions improve the sentence, indicate (d) as your response on the Answer Sheet. Thus a ‘No improvement’ response will be signified by the letter (d).




28. The Union Finance Minister has said that fresh recruitment will be restricted to one-third of the vacancies that arise in the government jobs:

(a) rise     (b) raise     (c) arouse     (d) No improvement



29. Our armed forces are superior to those of any other country in the world.

(a) superior than (b) superior from

(c) superior over (d) No improvement




30. He secured the first position in a hundred metres race:

(a) a hundred metre race

(b) hundred metres race

(c) one hundred metres race

(d) No improvement



31. Working in the slums brought her in against the realities of poverty.

(a) brought her forward

(b) brought her up


(c) brought her on

(d) No improvement



32. I hope it’s not imposing on your hospitality, but could I stay to dinner?

(a) stay over (b) stay up to

(c) stay at (d) No improvement



33. I have dreamt all my life for owning a beautiful maroon-coloured car.

(a) of owning (b) to owning


(c) at owning (d) No improvement



34. The war has had a negative impact over the economy of the country.

(a) in the economy (b) on the economy

(c) in economy (d) No improvement



35. They claimed to bring the best products and services on the doorsteps of their consumers.

(a) up to the doorsteps (b) to the doorsteps

(c) at the doorsteps (d) No improvement




36. My company has decided to go it alone rather than set up a joint venture.

(a) put up (b) deal with

(c) go along with (d) No improvement



37. The last exercise was fairly easier than I thought it would be:

(a) more (b) rather

(c) comparatively (d) No improvement



38. The Chief Manager asked me to carry on his orders immediately.

(a) carry forward (b) carry out


(c) carry with (d) No improvement



39. The profit will be dealt with among the investors:

(a) dealt in (b) dealt out

(c) dealt of (d) No improvement



40. Your advice is no different from the other friends.

(a) not different from

(b) not different from that of the

(c) not different from the


(d) No improvement



41. It was she, not me, who put forth the attractive proposition.

(a) she, not I, (b) her, not me,

(c) her, not I, (d) No improvement



42. His love of languages bent him towards a career as a translator.

(a) inclined him (b) twisted him

(c) directed him (d) No improvement




43. His suggestion was greeted with hoots of laughter.

(a) greeted in (b) greeted over

(c) greeted at (d) No improvement



ORDERING OF SENTENCES

Directions: In the following items each passage consists of six sentences. The first and the sixth sentence are given in the beginning. The middle four sentences in each have been removed and jumbled up. These are labelled P, Q, R and S. You are required to find out the proper sequence of the four sentences and mark accordingly on the Answer Sheet.



44. S1 : It is said that deep down people are alike. S6 : People differ in intelligence, personality, emotional display, values, priorities and the like. P : However in the broadest sense we can saypeople are all alike.

Q : This statement is essentially false.

R : But the individual differences are far more illuminating.


S : For instance, it’s true that people all have attitudes, likes and dislikes, feelings and similar attributes.



The proper sequence should be:

(a) Q R S P (b) S P Q R

(c) Q P S R (d) S R Q P



45. S1 : Let us look at the statement ‘It’s not what you say, but it’s what you do’.


S6 : But when words and actions diverge, people focus most on what they see in terms of behaviour.

P : Actions do speak louder than words.

Q : Words can influence others; we don’t deny.

R : The statement is mostly true.

S : This doesn’t mean that words fall on deaf ears.



The proper sequence should be:

(a) S Q R P (b) R P S Q


(c) S P R Q (d) R Q S P



46. S1 : There are no limit to insanitation.

S6 : So I asked for a broom to clean them myself.

P : There were only a few latrines

Q : They refused point-blank to clean them.

R : Pools of water were everywhere.

S : I pointed it out to the volunteers.



The proper sequence should be:


(a) R S P Q (b) Q P S R

(c) R P S Q (d) Q S P R



47. S1 : I did not feel at all sea-sick.

S6 : For I could rarely follow their remarks when they came up to speak to me.

P : I was quite unaccustomed to talking English, and except one all the other passengers in the second saloon were English.

Q : But as the days passed, I became fidgety.

R : I could not speak to them.

S : I felt shy even in speaking to the steward..




The proper sequence should be:

(a) P S Q R (b) Q R P S

(c) P R Q S (d) Q S P R



48. S1 : The hall darkened

S6 : And then father taught the girl arithmetic.

P : After all that, his father came on the screen.

Q : And then a little girl came up, and he spoke to her exactly as he used to speak to Sambu.

R : He was wearing just the dhoti and shirt he used to wear at home.


S : Sambu sat through the trailers and slide advertisements without enthusiasm.



The proper sequence should be:

(a) Q R P S (b) S P R Q

(c) Q P R S (d) S R P Q



49. S1 : Love for the country is a necessity.

S6 : God created the globe, but man drew lines of hatred and emmity on it.

P : Similarly nationalism has to be sacrificed at the altar of internationalism.

Q : But it should in no way exceed the limits and take the shape of jingoism.


R : Provincialism has to be sacrificed in the interest of the nation as a whole.

S : There is no reason why the nations of the world cannot live together with harmony and peace.



The proper sequence should be:

(a) P R Q S (b) Q S P R

(c) P S Q R (d) Q R P S



SELECTING WORDS/PHRASES

Directions: In the following passage at certain points, you are given a choice of three words/phrases, underlined and marked (a), (b), (c). Choose the best word/phrase out of the three and indicate your choice in the relevant column of your Answer Sheet.

50.


(a) acquired

(b) achieved

(c) obtained

freedom produces; and that cure is freedom. When a person first leaves his



51.

(a) prison

(b) barrack

(c) cell


he cannot



52. (a) bear

(b) stand

(c) tolerate

the light of day, he is unable to



53. (a) distinguish

(b) discriminate

(c) separate


or recognize faces. But the remedy is not to



54. (a) push

(b) throw

(c) remand

him to his



55. (a) quagmire

(b) dungeon

(c) precipice


but to



56. (a) adopt

(b) accustom

(c) reconcile

him to the



57. (a) rays

(b) twilight

(c) glare


of the sun. The blaze of truth and liberty at first



58. (a) glitters

(b) blazes

(c) dazzles

and bewilders nations which have become half blind in the house of



59. (a) prison.

(b) bondage.

(c) slaves.


But let them gaze on and they will soon be able to



60. (a) bear

(b) tolerate

(c) assimilate

it. In a few years men will learn to reason. The extreme violence of opinion


::: ANSWERS :::

1. (c) 2. (b) 3. (b) 4. (a) 5. (d) 6. (d) 7. (d) 8. (b) 9. (d) 10. (d) 11. (c) 12. (b) 13. (b) 14. (c) 15. (d) 16. (c) 17. (d) 18. (a) 19. (a) 20. (d) 21. (b) 22. (d) 23. (b) 24. (c) 25. (d) 26. (b) 27. (d) 28. (d) 29. (d) 30. (a) 31. (b) 32. (d) 33. (a) 34. (b) 35. (d) 36. (d) 37. (c) 38. (b) 39. (b) 40. (b) 41. (a) 42. (a) 43. (d) 44. (b) 45. (b) 46. (a) 47. (d) 48. (b) 49. (d) 50. (a) 51. (c) 52. (a) 53. (a) 54. (b) 55. (b) 56. (c) 57. (a) 58. (c) 59. (c) 60. (c)
****************************************************************
GIC_Quantitatative _Aptitude_SOLVED









Print Friendly and PDF
Share/Bookmark

0 comments:

Post a Comment

 

Recent Post

-::@ Disclaimer @::-

The blog content is all collected from source of internet and also my personal collection for helping students who are going to step into their new gen world where challenges are common and I love sharing and if anybody has any issues on this you can write to me . Coprights 2011-2012 @ www.36garhonline.blogspot.com

**HOPE FOR THE BEST
AND PREPARE FOR THE WORST**

Followers